Nothing Special   »   [go: up one dir, main page]

Quiz 1: Strategic Management and Strategic Competitiveness

Download as docx, pdf, or txt
Download as docx, pdf, or txt
You are on page 1of 59

Quiz 1: Strategic Management and Strategic Competitiveness

https://testbank45.com/wp-content/uploads/2018/06/9781133584698_TestBank_Chapter-1.pdf
1. Define strategic competitiveness and above-average returns. What is the relationship
between strategic competitiveness and returns on investment? (Xác định khả năng cạnh tranh
chiến lược và lợi nhuận trên trung bình. Mối quan hệ giữa năng lực cạnh tranh chiến lược và lợi
tức đầu tư là gì?)
Strategic competitiveness is achieved when the firm successfully formulates and implements a value-
creating strategy. Above-average returns are returns in excess of what investors expect to earn from
other investments with similar risk levels. Firms will only be able to earn above-average returns if
they develop a competitive advantage. Competitive advantage derives from a strategy that competitors
cannot duplicate or find too costly to imitate.
2. Hypercompetition is a characteristic of the current competitive landscape. Define
hypercompetition and identify its primary drivers. How can organizations survive in a
hypercompetitive environment? (Hypercompetition là một đặc điểm của bối cảnh cạnh tranh
hiện tại. Xác định siêu cạnh tranh và xác định trình điều khiển chính của nó. Làm thế nào các tổ
chức có thể tồn tại trong một môi trường siêu cạnh tranh?)
Hypercompetition is a condition of rapidly escalating competition based on price-quality positioning,
competition to create new knowledge and establish first-mover advantage, and competition to protect
or invade established product or geographic markets. In hypercompetition, firms aggressively
challenge their competitors. Markets are assumed to be inherently unstable and changeable. The two
primary drivers of hypercompetition are the global economy and rapid technological change. To
survive in a hypercompetitive environment firms need strategic flexibility. This demands continuous
learning which allows the firm to develop new skills so that they can adapt to the changing
environment and to consistently engage in change.
3. Describe the industrial organization (I/O) model of above-average returns. What are its
main assumptions? What is the key to success according to the I/O model?
The I/O model of above-average returns argues that the external environment is the primary
determinant of firm success, rather than the firm's internal resources. The model has 4 underlying
assumptions:
1) The external environment is assumed to impose pressures and constraints that determine the
strategies that would result in above-average returns.
2) Most firms competing within a particular industry, or in a certain segment of the industry, are
assumed to control similar strategically relevant resources and pursue similar strategies in light of
those resources.
3) Resources used to implement strategies are mobile across firms, which results in resource
differences between firms being short-lived.
4) Organizational decision makers are assumed to be rational and committed to acting in the firm's
best interests as shown by their profit maximizing behaviors.
The key to success according to the I/O model is to find the most attractive industry (the one with the
highest profit potential) in which to compete.
4. Describe and discuss the resource-based model of above-average returns.
The resource-based model focuses on the firm's internal resources and capabilities. These resources
and capabilities determine the firm's strategy and its ability to earn above-average returns. The firm's
resources are inputs into its production process. Resources must be formed into capabilities, the
capacity to perform a task or activity in an integrative manner. According to this model, capabilities
evolve over time and must be managed dynamically to achieve above-average returns. Resources and
capabilities that give a firm a competitive advantage are called core competencies. This model
assumes that resources are not highly mobile across firms; consequently, all firms within a particular
industry may not possess the same strategically relevant resources and capabilities. So, different firms
will have different core competencies. The organization's strategy is based on finding the best
environment in which to exploit its core competencies.
5. What are a firm's vision and mission? What is the value to the firm of having a specified
vision and mission?
The firm's vision is a picture of what it wants to be and what it wants to ultimately achieve. The firm's
mission is based on its vision. It specifies the business(es) in which the firm intends to compete and
the customers it intends to serve. The value of having a vision and mission is that they inform
stakeholders what the firm is, what it seeks to accomplish, and who it seeks to serve. A successful
vision is inspirational. The mission is more concrete and guides employees' behavior as they achieve
the firm's vision. Research shows that an effectively formed vision and mission positively impact firm
performance in terms of growth in sales, profits, employment, and net worth.
6. Describe an organization's various stakeholders and their different interests. Under
what condition can the firm most easily satisfy all stakeholders? If the firm cannot satisfy all
stakeholders, which ones must it satisfy in order to survive?
Stakeholders are the individuals and groups who can affect and are affected by the strategic outcomes
achieved and who have enforceable claims on a firm's performance. There are 3 principal types of
stakeholders:
1) Capital market stakeholders: the shareholders and the major suppliers of capital to the firm. They
are most interested in the return on capital in relation to the risk incurred.
2) Product market stakeholders: customers, suppliers, host communities, and unions representing
workers. The customers seek a reliable product at the lowest possible price. The suppliers seek loyal
customers willing to pay the highest sustainable price. Host communities want companies willing to
be long-term employers and providers of tax revenues. Union officials want secure jobs with good
working conditions for the workers they represent.
3) Organizational stakeholders: employees (managerial and non-managerial). These stakeholders
expect a firm to provide a dynamic, stimulating, and rewarding work environment. The firm can most
easily satisfy all stakeholders if it earns above average returns. If the firm does not earn above-average
returns, it must prioritize its stakeholders by their power, urgency, and degree of importance to the
firm. The firm must then make trade-offs among the stakeholders.
7. Who are the firm's strategic leaders? How do strategic leaders predict the profit
outcomes of different strategic decisions?
The firm's strategic leaders include the CEO and top-level managers, but they also include
organizational members who have been delegated strategic responsibilities. Strategic leaders use the
strategic management process to help the firm reach its vision and mission. Mapping an industry's
profit pool is one way strategic leaders can anticipate the profitability of different strategic decisions.
A profit pool is the total profits earned in an industry along all points in the value chain. This helps
the leaders determine where the primary sources of profit in the industry are located and allows them
to take actions to tap these sources.
8. Explain the relationship of the strategic management process to organizational ethics.
Almost all strategic management process decisions have ethical implications because they affect
stakeholders. The decision of the strategic leaders influence the organization's culture which is based
on the organization's core values (which are also influenced by the strategic leaders). The
organization's culture can be functional or dysfunctional, ethical or unethical. Consequently, the
strategic leader's role has a large impact on whether the organization is a good citizen.
9. What are the primary aspects of the strategic management process? You may reference
specific chapters from the text in formulating your response.
The strategic management process consists of three primary processes:
1) Analysis: involves the development of an understanding of the external environment and internal
organization
2) Strategy formulation: with knowledge about its external environment and internal organization, the
firm forms its vision and mission and makes decisions as to what strategies to utilize to provide
returns to shareholders
3) Implementation: putting the formulated plan into action
10. Explain why it is important for organizations to analyze and understand the external
environment.
Organizations do not exist in isolation. The external environment of the organization presents threats
and opportunities which the organization must address in its strategic actions. Parts of the
organization's external environment are changing rapidly, such as technology, and the organization
must constantly adjust to these changes. The information that the organization gathers about
competitors, customers and stakeholders is used to build the organization's capabilities or to build
relationships with stakeholders in the external environment. The information that the organization
gathers about the external environment must be matched with its knowledge of its internal
environment to form vision, to develop its mission, and to take actions that result in strategic
competitiveness and above-average returns.

Case Scenario 1: Palmetto. Palmetto was an early pioneer of personal data assistants (PDAs)
and dominates that market space (in terms of market share) with its core product, the Palmetto
Pidgy. Because this product category was entirely new to the market, Palmetto had to internally
develop the hardware and software sides of the business, and today is both a manufacturer of
PDAs and a programmer and licensor of its PDA operating system software. Recently, however,
the hand-held device maker's performance has taken a dive as a result of slumping sales and
costly inventory problems. New large entrants are entering both the equipment and software
sides of its business, putting further pressure on margins. Management is currently considering
its options, including the break up of Palmetto into two separate, independent public companies
- one devoted to hardware, the other software. -(Refer to Case Scenario
1) What primary business strategy issues does Palmetto face?
Recognizing that students have only just been introduced to strategy in this introductory chapter, the
Palmetto scenario helps frame and contrast the basic business and corporate strategy questions. The
best answers to the first question will start by noting that Palmetto appears to be in two distinct
businesses, hardware and software, which in turn are likely to have very different success factors and
competitors. Students can then begin talking about these competitors and the potential resources they
bring to the table (for instance, Microsoft in software and Sony in miniaturized consumer electronics).
This scenario also leads to a natural discussion of the attractiveness of the PDA market, and where the
most money is likely to be made.
2) What primary corporate strategy issues does Palmetto face?
Since the business strategy question should have revealed that Palmetto is actually in at least two
distinct businesses, the best answers to the corporate strategy question will begin by assessing which
of the businesses is more attractive, and whether or not Palmetto needs to be in both to compete, or
should specialize in either software or hardware. Companies which are diversified will have a
corporate strategy that encompasses various businesses with different business strategies. Students
can be prompted to debate the tradeoffs between retaining both businesses versus breaking the
company in two -a useful role play exercise entails asking students to walk through the likely resource
allocation tradeoffs that the diversified Palmetto must currently make

2) How do the I/O and resource-based models help you make recommendations to Palmetto's
management regarding a split into two companies? Do they lead to the same recommendation?
The best answers will begin by noting that the two models should be viewed as complementary and
applied in an integrative manner. Since the perspectives are complementary, the choice of I/O or
resource-based perspective as a starting point is simply a matter of taste. For instance, the discussion
can then flow to how the I/O perspective will help management understand the characteristics of the
two basic industries in which it participates (hardware and software), and perhaps lead to insights into
what factors allow one firm to compete effectively against other industry incumbents. The resource-
based model can then be applied to develop an understanding of where Palmetto is strongest in terms
of resources, capabilities, and core competencies. Further industry analysis can show whether or not
these resources will likely lead to competitive advantage in their respective markets. Through the
combination of these two perspectives, students can then help management determine whether
Palmetto can afford to remain a diversified firm or if it can only compete effectively by focusing on
either its hardware or software business

Case Scenario 2: Jewell Company. Jewell Company is a diversified manufacturer and marketer
of simple household items, cookware, and hardware. In its annual report, it expresses its
strategy as follows: "Jewell manufactures and markets staple volume lines to the volume
purchaser. We aim to increase shareholder value by continuing to build a company with
superior earnings per share growth and return on investment (ROI), and to earn a reputation
for excellence in performance and management. We plan to do this by merchandising to the
customer goods market a multi-product offering with superior customer service performance
for maximum market leverage. Through this we will achieve an ROI of 20% plus EPS growth
of 15%, with the constraint that debt not exceed half of our equity." -(Refer to Case Scenario 2)
1) What are the strengths and weaknesses of the above strategy statement?
Good strategies help managers to make tough decisions, which necessarily require them to
make tradeoffs. In terms of strengths, the best answers will note that Jewell’s strategy is fairly precise
with regards to target customers (volume purchasers like Walmart), product characteristics (staples,
manufactured items),how the firm plans to win the customers’ business (merchandising and multiple-
product offering), and measurable performance objectives. In terms of weaknesses, staple
manufactured products encompass a multitude of potential products (pens to hairbrushes to curtain
rods) so the statement does not signal any particular type of manufacturing expertise. The statement
also does not tellus how the firm plans to grow (internal means versus acquisition).
Finally,management has stated a growth goal, not a growth plan. Absent a clear plan,strategic
compromises and inconsistencies in the pursuit of growth will likely erode the competitive
advantage that Jewell had with its original varieties of product offerings and target customers.
2) Which groups of stakeholders does Jewell's statement appear to speak to?
This statement focuses on capital market stakeholder groups and one product market group, the
customers. This is because Jewel manufactures and markets staple volume lines to the volume
increase. They want their product get high demand from customer as well as customer has many
products to choose from then get the increasing in ROI. Customers are become their focuses to speak
to and then maximum their market leverage.
3) Does Jewell Company's statement of strategy include a vision statement or a mission
statement? Why or why not?
Strategic intent is internally focused. It is concerned with identifying the resources,
capabilities, and core competencies on which a firm can base its strategic actions. While
strategic mission is a statement of a firm's unique purpose and the scope of its operations in
product and market terms. Strategic Mission flows from strategic intent. It is externally focused.It
describe strategic intent and strategic mission when Jewell mention that they want to aim to increase
shareholder value by continuing to build a company with superior earnings per share growth
and ROI, and to earn reputation for excellence in performance and management. Besides that
Jewell also said that they plan to do the merchandising to the customer good market a multi-product
offering with superior customer service performance for maximum marketleverage.

Case Scenario 3: Vivendi Universal SA. Vivendi Universal is a French firm that started in 1853
as Companie General des Eaux. It grew from a French water utility company into one of the
world's largest conglomerates. Under the corporate leadership of CEO Jean Marie Messier,
Vivendi Universal became a highly diversified company involved in music, publishing, film, pay
TV, telecoms, Internet, water distribution, thermal energy supply, building and heavy public
construction projects, waste management, electrical energy services, real estate and other
activities. Mr. Messier was forced out of the company in July, 2002, in a liquidity crisis and
mounting shareholder anger. The acquisitions made by Mr. Messier saddled the company with
billions of dollars of debts. Vivendi shares plummeted 80 percent during the last six months Mr.
Messier was CEO, according to the Wall Street Journal. Meanwhile, the SEC indicated that a
disputed severance payment of $23 million to Mr. Messier may actually constitute "ill gotten
gains," reported the Wall Street Journal. Vivendi Universal refused to make the payment
saying the board and shareholders had never agreed to the severance payment. On the brink of
bankruptcy, Vivendi Universal brought in Jean-Rene Fourtou to replace Mr. Messier as CEO.
According to the business media, Mr. Fourtou has taken a dying enterprise and given it a
survival plan. He sold numerous Vivendi Universal businesses, bringing the company to focus
on Cegetel, a phone company; SFR, a cellphone company; Canal Plus, a television company;
and Universal Music. Mr. Fourtou was able to reduce Vivendi's debt from 37 billion euros in
2002 to a projected 5 billion euros by the end of 2005. The company showed its first quarterly
profit at the end of 2003, allowing Mr. Fourtou to arrange a loan from a banking consortium
and give the company hopes that credit-rating agencies would raise its debt from junk-bond
status, according to The New York Times. The company projects $2.2 billion in profits for 2005.
According to Barrons, "Fourtou and the new chief executive Jean Bernard Levey have moved
beyond restructuring and recapitalizing the firm to building core businesses." -(Refer to Case
Scenario 3)
1) Who are stakeholders of Vivendi Universal, and what was the role of stakeholders in
Vivendi Universal's recent history?
Stakeholders are the individuals and groups who can affect, and are affected by, the strategic
outcomes achieved by the firm and who have enforceable claims on a firm’s performance.
Stakeholders support an organization as long as its performance meets or exceeds their expectations.
As Vivendi Universal grew ever larger and more diverse, its financial performance declined. Thus it
lost the support of its capital market stakeholders, its shareholders. They rebelled, and the result was
the firing of Mr. Messier, the instigator of the growth, and the installation of Mr. Fourtou, who
immediately began divesting most of the companies Mr. Messier had purchased. A second
stakeholder group was the individuals and organizations holding Vivendi Universal debt (bonds and
bank loans) that were threatened by the impending bankruptcy of the firm. Mr. Fourtou pleased this
group of stakeholders by reducing the debt of the firm by selling off the excess companies. Finally,
shareholders were also supported by the SEC, which is investigating the multi-million severance
payment to Mr. Messier.
2) Who was ultimately responsible for the problems at Vivendi Universal?
Some believe that every organizational failure is actually a failure of those who hold the final
responsibility for the quality and effectiveness of a firm’s decisions and actions. Strategic leaders are
the people responsible for the design and execution of strategic management processes. At Vivendi
Universal, Mr. Messier, the former CEO, seems to have borne the brunt of public blame. But
Vivendi’s top management team and the board of directors must assume some of the blame because
Mr. Messier did not act alone. The pivotal role that can be played by a CEO as a strategic leader is
also illustrated by the successful changes instituted by Mr. Fourtou, the new CEO.

Quiz 2: The External Environment


https://wenku.baidu.com/view/c48d2116f18583d0496459c7.html
https://quizlet.com/28178805/strategic-management-case-scenario-application-questions-
flash-cards/
https://testbankplus.com/sample/pdf/9781285425177-TEST-BANK.pdf
1. Identify and describe the three major parts of the external environment. What is the
purpose of the firm's collecting information about these aspects of its environment?
The external environment has three major parts. The first is the general environment, which is
composed of dimensions in the broader society that affect industries and their firms. These
environmental segments are: demographic, economic, political/legal, sociocultural, technological, and
global. The second part of the external environment is the industry environment, which involves five
factors that influence a firm, its competitive actions and responses, and the industry's profit potential.
These five factors are: the threat of new entrants, the power of suppliers, the power of buyers, the
threat of product substitutes, and the intensity of rivalry among competitors. The competitor
environment is the third part of the external environment. The firm must be able to predict
competitors' actions, responses, and intentions. With the information collected about these aspects of
its external environment, the firm can develop its vision, mission, and strategic actions.
2. Describe and discuss the four activities of the external environmental analysis process.
The external environmental analysis process includes four steps: scanning, monitoring, forecasting,
and assessing. The scanning of the environment includes the study of all segments of the general
environment in order to detect changes that may occur in the future or that already are occurring. This
is critical in a volatile environment. Scanning often deals with ambiguous, incomplete, or
unconnected data and information. When analysts monitor the environment, they observe
environmental changes to see if an important trend is emerging from those spotted by scanning. It is
critical for the firm to detect meanings in these events and trends so that it can be prepared to take
advantage of opportunities these trends provide. Forecasting builds on scanning and monitoring to
develop feasible projections of what might happen and how quickly it will occur. Forecasting is
important in helping the firm adjust sales to meet demand. Finally, through assessing, the analyst
determines the timing and the significance of the effects of environmental changes and trends on the
strategic management of the firm. Assessment must specify the competitive relevance of the data
3. Describe the seven segments of the general environment.
1) The demographic segment encompasses factors such as population size, geographic distribution,
age structure, ethnic mix, and income distribution.
2) The economic segment involves the nature and direction of the economy in which a firm competes
or may compete, domestic as well as global.
3) The political/legal segment is the arena in which organizations compete for attention, resources,
and a voice in laws and regulations guiding the interactions among nations.
4) The sociocultural segment is concerned with society's attitudes and cultural values.
5) The technological segment includes institutions and activities involved with creating new
knowledge and transforming it into new outputs, products, processes, and materials.
6) The global segment includes new global markets, existing markets that are changing, international
political events, and critical cultural and institutional characteristics of global markets.
7) The physical segment includes potential and actual changes in the physical environment (such as
global warming) and business practices that are intended to positively deal with those changes (such
as control of carbon emissions and other environmentally friendly actions).
4. What are high exit barriers and how do they affect the competition within an industry?
Exit barriers are economic, strategic, and emotional factors causing companies to remain in an
industry, even though the profitability of doing so is in question. The following are common sources
of exit barriers: 1) specialized assets which cannot be used in another business or location; 2) fixed
costs of exit, such as labor agreements which penalize a firm for ceasing operation; 3) strategic
interrelationships or mutual dependence of business units wherein one business of a corporation
serves another corporate business; 4) emotional barriers that cause owners to be sentimentally
attached to the business or to their own role in it; 5) governmental and social restrictions that prevent
a firm from closing, often in order to prevent the loss of jobs in a country or community. \

5. Identify the five forces that underlie the five forces model of competition. Explain briefly
how they affect industry profit potential.
1) Threat of new entrants: New entrants threaten existing firms' market share. They increase
production capacity in an industry which results in lower profits for all firms, unless demand is
increasing. The new entrant may force the existing firms to be more effective and efficient in
production, and to compete on new dimensions.
2) Power of suppliers: Suppliers with high power can increase prices and decrease the quality of their
products sold to the firm. If firms are unable to pass along price increases to customers, their profits
diminish.
3) Power of buyers: When buyers (customers) have high power they can force prices down, and
require increases in quality and service
levels, thus driving profits down.
4) Substitutes: Substitutes perform the same or similar functions of the firm's product. The price of the
substitute places an upper limit on prices firms can charge for the original product, limiting industry
profits.
5) Intensity of competitive rivalry affects the firm's ability to make a profit as competitors' actions
challenge the firm or competitors try to improve their market position. Increasing rivalry reduces the
ability of weaker firms to survive.
6. What is a firm's strategic group? What effect does the strategic group have on the firm?
The firm's strategic group is the set of firms that emphasize similar strategic dimensions and use a
similar strategy. The firms in a strategic group occupy similar positions in the market, offer similar
goods to similar customers, and may make similar decisions about production technology and
organizational features. Competition among firms in a strategic group is more intense than the
competition among a firm and those firms outside its strategic group. Actions of members in the
firm's strategic group affect its strategic decisions in many areas including pricing, product quality,
and distribution.
7. What do firms need to know about their competitors? What legal and ethical
intelligence gathering techniques can be used to obtain this information?
Competitor analysis helps firms identify:
1) what drives the competitors by understanding the competitor's FUTURE objectives);
2) what the competitor is doing and is capable of doing by understanding the competitor's CURRENT
STRATEGY;
3) what the competitor believes about the industry by understanding the ASSUMPTIONS made by
the competitor; and
4) what the competitor's CAPABILITIES are by understanding the competitor's strengths and
weaknesses. Firms can legally and ethically gather public information, such as annual reports, SEC
reports, UCC filings, court records, and advertisements. Firms can also attend trade fairs to obtain
competitors' brochures, view exhibits, and discuss products. This data combines to form competitive
intelligence.
8. What are the barriers to entry and how do they affect competition?
Entry barriers discourage competitors from entering a market and facilitate a firm's ability to remain
competitive in a market in which it currently competes. Barriers to entry include:
1) Economies of scale are derived from incremental efficiency improvements through
experience as a firm grows larger.
2) Product differentiation occurs when over time, customers may come to believe that a firm's product
is unique. This belief can result from the firm's service to the customer, effective advertising
campaigns, or being the first to market a product or service.
3) Capital requirements – Competing in a new industry requires a firm to have resources to invest. In
addition to physical facilities, capital is needed for inventories, marketing activities, and other critical
business functions.
4) Switching costs are the one-time costs customers incur when they buy from a different supplier.
5) Access to distribution channels – Over time, industry participants commonly learn how to
effectively distribute their products. Once a relationship with its distributors has been built a firm will
nurture it, thus creating switching costs for the distributors.
6) Cost disadvantages independent of scale – Sometimes, established competitors have cost
advantages that new entrants cannot duplicate. Proprietary product technology, favorable access to
raw materials, desirable locations, and government subsidies are examples.
7) Government policy – Through their decisions about issues such as the granting of licenses and
permits, governments can also control entry into an industry.

Case Scenario 1: The Boys and Girls Club. The Boys and Girls Club (BGC) is a national non-
profit organization geared to provide America's youth with the tools and skills they need to
become healthy adults, responsible citizens, and effective leaders. By bringing parents,
neighbors, educators, and civic leaders together with our youth, BGC believes it can instill these
crucial life lessons at an age when they're most needed. The national organization is
headquartered in Atlanta, GA, and serves as a service hub for over 3,700 club locations around
the U.S. Each local club is directed by a volunteer board of directors and staffed by professional
youth development workers (usually including an executive director, a program director, and
an arts director) and many volunteers who just enjoy working with young people and want to
make a difference in their lives. While affiliated with the national center, each local BGC is
locally funded. -(Refer to Case Scenario 1)
1. How are the various facets of the general environment (Table 2.1 in Strategic
Management) likely to be important for BGC?
The best answers will begin by noting that BGC has a mission focused on the education and social
development of needy youth. Thus, the demographic, economic, sociocultural. and physical segments
may be the segments of primary importance. Within the physical segment, for instance, BGC may
consider what it can do to respond to climate change and depletion of energy resources. The global
segment is also a natural discussion point since contexts far from home may not come to our attention
until after a critical stage has been passed. For instance, the presence of immigrants and refugees in a
community affect the needs of BGC's clientele.
2. Why would attention focused on victims of natural disasters be a threat to the BGC?
The best answers will observe that BGC is entirely dependent upon local donations for its operations
and public focus on other causes will likely draw away donation dollars that had been historically
earmarked for BGC. This alternative charitable giving serves donors as a substitute for donations to
BGC.
3. How might the BGC respond to threats to their donations at both local and national
levels?
Since BGC is governed locally by a board of directors drawn from the community, the local
organizations should use these members to rally support against their dwindling donation base. The
board and BGC staff members can also reach out to other local organizations and community
govermments. At a national level, image ads and the lobbying of various national organizations
(government, teachers' associations, minority outreach organizations, environmental groups, etc.) can
be initiated and managed through the BGC headquarters in Atlanta.
1. Get more media support (newspapers, magazine)
2. Provide testimony and evidence that project is working
3. Competition - to create awareness
4. Centralization of national entity for donation

Case Scenario 2: B.B. Mangler. B.B. Mangler is a top U.S. business-to-business distributor of
maintenance, repair, and service equipment, components, and supplies such as compressors,
motors, signs, lighting and welding equipment, and hand and power tools. Its industry is
typically referred to as MRO, which is an acronym for maintenance, repair, and supplies. MRO
products are typically small, fairly inexpensive (light bulbs and washers), but often needed on
short notice. It states its strategy as having the "capacity to offer an unmatched breadth of
lowest total cost MRO solutions to business." Mangler's GoMRO sourcing center for indirect
spot buys locates products through its database of 8,000 suppliers and 5 million products.
Mangler has 388 physical branches in the U.S., including Puerto Rico (90% of sales), 184 in
Canada, and 5 in Mexico. Customers include contractors, service and maintenance shops,
manufacturers, hotels, governments, and health care and educational facilities. Mangler also
provides materials-management consulting services. -(Refer to Case Scenario 2)
1. Historically, Mangler appears to have relied on its physical locations for market
presence in the U.S. and northern South America. What threats does the Internet pose
to its location-based strategy?
The best answers will start by noting that Mangler's location-based strategy is also likely to require
quite a bit of investment in inventory (keeping all those parts on hand at each of its branches in the
United States, Canada, and Mexico). Given that it competes in a low-cost industry. and itself
competes on cost. an Internet-based MRO competitor may be able to create an even lower cost
structure (as Amazon.com did with books). The Internet seems like a natural fit for the MRO market.
Such an online strategy may be particularly effective for those MRO items that are less time-critical.
2. What opportunities does the Internet provide to Mangler, both domestically and
internationally?
Answers to this question should suggest several different responses to the ways in which Mangler
could capitalize on the Internet domestically. The best answers for the international strategy question
will begin by noting that just as Mangler's many domestic locations provide a barrier to entry in its
markets by potential competitors (that is, it already has the market share to cover its high physical
location costs and also is likely to have tremendous goodwill). so too have they been a barrier against
Mangler's entry into other international markets such as Europe, Asia, and other parts of Latin
America. The Internet does away with this barrier to a great extent, which levels the playing field
between Mangler and the incumbents of those respective international markets.
3. How should Mangler respond to the threat of new Internet-based entrants?
There are several possible avenues, and the best answers will note these alternatives. The most
obvious response would be for Mangler to start up a web-based complement to its location-based
delivery system. A related response might involve the centralization of low-demand, high-cost items
to specific areas of the country, where they could then be funneled rapidly to the actual local outlets
using the Internet as an internal market. Finally, Mangler could hedge this threat by investing in the
most promising online rivals.

Case Scenario 3: Barracuda Inc. Barracuda Inc. is a lamp fixture manufacturer that is
considering an entry strategy into the U.S. home furnishings manufacturing industry. The
existing landscape consists of many players but none with a controlling share. There are
presently 2500 home furnishings firms, and only 600 of those have over 15 employees. Average
net profit after tax is between 4 and 5%. While the industry is still primarily comprised of
single-business family-run firms that manufacture furniture domestically, imports are
increasing at a fairly rapid rate. Some of the European imports are leaders in contemporary
design. Relatively large established firms are also diversifying into the home furnishings
industry via acquisition. Supplier firms to the home furnishings industry are in relatively
concentrated industries (like lumber, steel, and textiles). Retailers, the intermediate customer of
the home furnishings industry, have been traditionally very fragmented. Customers have many
products to choose from, at many different price points, and few home furnishing products have
strong brands. Also, customers can switch easily among high and low-priced furniture and
other discretionary expenditures (spanning big screen TVs to the choice of postponing any
furniture purchase entirely). -(Refer to Case Scenario 3)
1. Using the five-forces framework, summarize the opportunities and threats facing
Barracuda as it considers entry into the home furnishings manufacturing industry.
Which threats are greatest to current incumbents?
The best answers will be based on an application of the five forces model to the scenario, From this
model students should be able to point out that the most significant threats are the power of
consumers, lack of economic power with suppliers, and increasing presence of imports. These
characteristics plus the highly fragmented nature of the industry itself are likely to translate into near-
perfect competition, leaving no single player with a clear advantage. Opportunities may exist in
particular niches, depending on the internal strengths of new entrants. In terms of the larger market,
there appears to be an opportunity for a large firm to consolidate the industry and add brand power,
thereby potentially gaining power over suppliers and customers.
2. How intense is competitive rivalry likely to be among incumbents of the home
furnishings manufacturing industry?
The best answers will be able to walk through the determinants of rivalry spelled out in pages 57
through 59. The fact that this industry is fairly characterized as having nearly perfect competition
suggests that rivalry is high. Larger players are likely to have significant exit barriers. particularly
given the slow growth. high fixed costs, lack of differentiation, and low profitability of the market
overall. Thus, new larger entrants to this industry may further escalate the degree of competition.
3. Is the furniture industry described above attractive?
Astute students may begin by noting that this industry is attractive if you are in a position that is
currently less attractive than that demonstrated by the home-furnishings business. Beyond that,
discussion should generally lead to the recognition that this industry is currently unattractive -
summarized by its paltry profit margins, fragmented membership. lack of power over suppliers and
customers, and high degree of rivalry.

Quiz 3: The Internal Organization


https://wenku.baidu.com/view/9f32b80503d8ce2f006623c7.html
1. Describe the importance of internal analysis to the strategic success of the firm.
By analyzing its internal environment, a firm determines what actions it can take based on its unique
resources, capabilities and core competencies. The firm's core competencies are the source of the
firm's competitive advantage. Internal analysis allows the firm to compare what it is capable of doing
(what it "can do") with what it "might do" (which is a function of opportunities and threats in the
external environment). Matching what a firm can do with what it might do allows the firm to develop
its vision, pursue its strategic mission, and select and implement its strategies. This allows the firm to
leverage its unique bundle of resources and capabilities to gain competitive advantage.
2. What are the differences between tangible and intangible resources? Which category of
resources is more valuable to the firm?
Resources are either tangible or intangible. Tangible resources are those assets that can be observed
and quantified. There are 4 types of tangible assets:
1) financial resources (borrowing capacity, ability to generate internal funds);
2) physical resources (plant and equipment, access to raw materials);
3) technological resources (patents, trademarks, copyrights, and trade secrets); 4) organizational
resources (formal reporting structure, planning, controlling and coordinating systems).

Intangible resources are those assets in the firm that are less visible. There are 3 types of such
resources:
1) human resources (knowledge, trust, management capabilities, and organizational routines),
2) resources for innovation (ideas, scientific capability, and capacity for innovation), and
3) reputation (reputation with customers, i.e., the firm's brand name and perceptions of product
quality, and relationships with suppliers). Intangible assets develop over time and are deeply rooted in
the organization's history. Consequently, they are difficult for competitors to analyze and imitate. In
addition, intangible resources can be leveraged to create new value to the firm.

These properties give intangible resources a greater ability to create sustainable competitive
advantage than do tangible resources.
3. Define capabilities and how they affect the firm's strategic success.
Capabilities exist when resources have been purposely integrated to achieve a specific task or tasks.
Examples: HR activities, product marketing, R&D.
Capabilities are based on developing, carrying, and exchanging information and knowledge through
the firm's human capital. Many of the firm's capabilities are based on the unique skills and knowledge
of its employees and their functional expertise. The knowledge possessed by human capital is among
the most significant of a firm's capabilities. Capabilities are often developed in specific functional
areas or in part of a functional area.
4. Describe the four specific criteria that managers can use to decide which of their firm's
capabilities have the potential to create a sustainable competitive advantage.
Managers must identify whether their firm has capabilities that are valuable and nonsubstitutable from
the customer's point of view, and unique and inimitable from the firm's competitors' point of view.
Only capabilities with these four characteristics are core competencies that can lead to sustainable
competitive advantage:
1) Valuable
2) Rare
3) Costly-to-imitate
4) non-substitutable
5. Describe a value chain analysis. How does a value chain analysis help a firm gain
competitive advantage?
A value chain analysis allows a firm to understand the activities that create value for the firm and
those that do not. A value chain follows the product from its raw-material stage to the final customer.
The purpose is to add as much value as possible as cheaply as possible and to capture that value. To
conduct a value chain analysis, managers should study and identify all activities of the firm and
evaluate their impact on the effort to create value for the customer. Two central activities in a value
chain:
1) Primary activities: involved in a product's physical creation, its sale and distribution, and its service
after the sale
2) Support activities: necessary for the primary activities to take place
6. Why is it important to prevent core competencies from becoming core rigidities?
All core competencies have the potential to become core rigidities and to generate failure. Each
competence is a potential weakness if it is emphasized when it is no longer competitively relevant.
The success that the competence generated in the past can generate organizational inertia and
complacency. A core competence can become obsolete if competitors figure out a better way to serve
the firm's customers, if new technologies emerge, or if political or social events shift in the external
environment. If the organization's managers react to these changes with inflexibility and strategic
myopia, then core rigidities are created.
7. What is value? Why is it important?
Value measured by a product's performance characteristics and by its attributes for which customers
are willing to pay.
8. Define outsourcing. Why do organizations outsource?
The purchase of a value-creating activity or a support function activity from an external supplier.
9. Why is it important to identify internal strengths and weaknesses?

10. Describe an organization with which you are familiar. Does it have a sustainable
competitive advantage?

Case Scenario 1: Heartsong LLC. Heartsong LLC is a designer and manufacturer of


replacement heart valves based in Peoria, Illinois. While a relatively small company in the
medical devices field, it has established a worldwide reputation as the provider of choice high-
quality, leading-edge artificial heart valves. Most of its products are sold to large regional
hospital systems and research hospitals. Specialty heart centers are another emerging, but fast-
growing, market for its valves. While Heartsong would like to grow quickly, its growth is
constrained by the need to finance larger production runs and then carry this additional
inventory. For products like those of Heartsong, vendors typically do not collect payment until
the unit is actually used in surgery. Moreover, heart valves are usually required on short notice
which means that they must be either onsite, or inventoried at a nearby location. If nearby, then
transport of the unit to a hospital or heart center occurs within a matter of hours, and
sometimes minutes. For this reason, accelerated growth would require Heartsong to both
finance increased production of its heart valves, along with carrying increased levels of
inventory that are in fact sitting on their customers' shelves. In fact, inventory-carrying cost is
its single largest cost outside of research and development. While profitable growth is necessary
if Heartsong is to continue extending its competitive advantage through increasingly greater
investments in basic heart valve R&D, it is not clear that the company can internally support all
these increased financial commitments (R&D, manufacturing, and inventory). Doc Watson, the
CEO of Heartsong, is considering an outside contractor, EdFex, to handle the inventorying,
warehousing, and delivery of its valves. EdFex has secure, high-tech warehouses in most major
population centers around the country, and can ensure delivery of a product to these markets
from its warehouses in less than one hour. -(Refer to Case Scenario 1)
1. What value-chain activities appear to underlie Heartsong's competitive advantage?
The best answers will begin by noting that Heartsong has the capacity to design leading-edge medical
products and then take these designs and turn them into reliably manufactured, high-quality
replacement heart valves. Thus, basic R&D and quality precision manufacturing are likely to be
critical value-creating facets for this firm
2. Why might an outsourcing arrangement with EdFex be attractive to Heartsong?
The best answers will start by observing that the scenario suggests that Heartsong needs to grow if it
is going to continue being competitive and successful. However, Heartsong is also capital constrained
and an outsourcing arrangement with EdFex allows it to more efficiently manage this significant
aspect of its cost base (inventory and delivery). This outsourcing solution would be ideal if it would
allow Heartsong to maintain a centralized warehouse with heart valve inventory in major population
centers, instead of its present practice of carrying inventory on the shelves of each of its hospital
customers. As a result, Heartsong could grow its market presence, while more efficiently managing
the need to have heart valves available on short notice.
3. What are the implications of an EdFex outsourcing arrangement for the capabilities
underlying Heartsong's competitive advantage?
The best answers will develop the theme that the EdFex outsourcing arrangement is truly likely to be
win-win. With the arrangement in place, Heartsong is able to devote its financial, human capital, and
managerial resources to basic R&D and quality precision manufacturing; and, EdFex does what it
does best in logistics. Moreover, it is hard to contemplate that EdFex would ever think of entering the
heart valve industry - thus, EdFex does not pose a direct threat as a future competitor. It does however
pose an indirect threat to Heartsong to the extent it can hold the firm hostage, and extract exorbitant
fees for its logistic services.
Case Scenario 2: ERP Inc. ERPI is a leading provider of enterprise integration software (EIS).
EIS allows a firm to connect and integrate processes across all aspects of its business, regardless
of where they are located around the world. ERPI is a product-focused company, whereas most
competitors in its market space, like Oracle, operate as "solutions companies." Oracle and
Microsoft have begun to devote considerable resources to the development of and acquisition of
products to compete in the EIS space. Despite these recent threats, one benefit of its product-
focused strategy is that ERPI's proprietary product is generally recognized as being 200% to
300% better than competitors' software. ERPI estimates it will take 2 to 3 years for competitors
to develop the capabilities needed to bring a competing product to market. ERPI invests a
considerable percentage of its profits in basic R&D to support its core products. As evidence of
this, among its competitors the firm maintains the largest in-house programming staff dedicated
solely to the development of advanced enterprise integration software. Installation and related
consulting for EIS typically cost between $100 and $200 million, with the ERPI software
component accounting for about 20% of the installed cost (the remaining 80% is spent on the
actual installation, not counting the value of the customer's time). ERPI's target market consists
of the world's largest manufacturing and industrial firms and it currently enjoys a 60 percent
market share. -(Refer to Case Scenario 2)
1. How valuable, rare, costly to imitate, and non substitutable are ERPI's capabilities?
The best answers will simply walk through the respective columns in Table 3.5 and reach the
conclusion that, at least in the near term, ERPI has a sustainable competitive advantage. Its EIS
software is valuable given that it is 200% to 300% better than competitors’ products. It is similarly
rare and nonsubstitutable since it is proprietary, and currently has a two-year lead on the alternatives.
A similar rationale can be invoked to support the argument that ERPI’s capabilities in software
programming are going to be costly to imitate. A competitor would have to hire a similar workforce
or acquire a company that currently occupies the same market space. This strong position is further
bolstered by the fact that a large percentage of the market is voting with its feet in favor of ERPI.
2. How sustainable is ERPI's competitive advantage?
The best answers will build on the basic notions developed in response to question 4. Students will
argue that ERPI’s competitive advantage is sustainable as long as its technology continues to define
the leading edge of EIS products and that substitute solutions do not encroach much on its two-year
lead. However, and as is consistent with most high-technology markets, as students pick apart ERPI’s
capabilities following the categories in Table 3.5 they should begin to see that sustained competitive
advantage in this particular market space may be difficult, particularly given the presence of large,
aggressive competitors like Oracle and Microsoft, which are intent on gaining a presence in the EIS
market.
3. Imagine that ERPI's historic growth strategy has focused on making one sale and then
moving on to the next target company. After several years of building market share
using this approach, what new resources has ERPI developed?
This question asks students to take a more dynamic perspective of potentially valuable resources that
companies and their customers create together, but that the company itself can exploit (a perfect
example of a co-specialized asset). The best answers will begin by observing that if ERPI has focused
historically on transactions (making the sale), then it has given little explicit consideration to
customers as long-term relationships beyond the need to provide technical support (lifetime value of a
customer beyond the first sale). Shifting attention to ERPI installations as relationships suggests that
the company now has a customer list to die for. This list is especially valuable since (1) the target
companies have invested upwards of $200 million in ERPI proprietary systems and, (2) once
installed, given the pervasive nature of EIS systems, those target firms are unlikely to simply switch
to another system.
Quiz 4: Business-Level Strategy
https://quizlet.com/272280883/mgmt-4900-midterm-flash-cards/
https://www.studyblue.com/notes/note/n/chapter-4-business-level-strategy/deck/7118779
https://tbmirate.com/product/strategic-management-concepts-and-cases-
competitiveness-and-globalization-11th-edition-by-michael-a-hitt-test-bank/
1. Define strategy and business-level strategy. What is the difference between these two
concepts?
In general, a strategy consists of the choices an organization makes in an attempt to gain strategic
competitiveness and earn above-average returns. The organization's strategic choices are influenced
by threats and opportunities in the external environment and by the nation and quality of its internal
re-sources, capabilities, and core competencies. The strategy reflects the firm’s vision and mission.
Business-level strategy is concerned with a particular product market. Business-level strategy is an
integrated and coordinated set of commitments and actions the firm uses to gain a competitive
advantage in a particular product market. It is the organization's core strategy. Every firm, no matter
how small, will have at least one business-level strategy. A diversified firm will have several types of
corporate-level strategies as well as a separate business-level strategy in each product market area in
which the company competes. The essence of a firm’s business level strategy is choosing to perform
activities differently or to perform different activities than competitors.
2. When a firm chooses a business-level strategy, it must answer the questions "Who?
What? and How?" What are these questions and why are they important?
- Who: Determining the customers to serve. It is important to know your market segments. This
allows you to tailor your strategy to their specific needs
- What: Determining which customer needs to satisfy. Customer needs are related to a product's
benefits and features. Fulfilling customer needs is how firms create value.
- How: Determining core competencies necessary to satisfy customer needs. Only firms with capacity
to continuously improve, innovate and upgrade their competencies can expect to meet and/or exceed
customer expectations across time.
The firm must decide (1) who are the customers who will be served, (2) what needs do the target
customers have that must be satisfied, and (3) how will those needs be satisfied by the firm. The
choice of target customer (who) usually involves segmenting the market to cluster people with similar
needs into groups. The target customers’ needs drive “what” benefits and features the firm’s product
will have. This involves a choice and balance between cost and differentiation of the product. Finally,
firms use their core competencies (how) to implement value-creating strategies and satisfy customers’
needs.
3. Discuss how a cost leadership strategy can allow a firm to earn above average returns in
spite of strong competitive forces. Address each of the five competitive forces.
A firm focuses on a niche market, adding value by leveraging value chain activities that allow value
creation through the cost leadership strategy.
• Competitive advantage: low cost
• Competitive scope: narrow industry segment
Rivalry: Having the low-cost position serves as a valuable defense against rivals. Because of the cost
leader's advantageous position, especially in logistics, rivals cannot reduce their costs lower than the
cost leaders', and so they cannot earn above-average returns.
Buyers: The cost leadership strategy also protects against the power of customers.Powerful customers
can drive prices lower, but they are not likely to be driven below that of the next-most-efficient
industry competitor. Prices below this would cause the next-most-efficient competitor to leave the
market, leaving the cost leader in stronger position relative to the buyer.
Suppliers: The cost leadership strategy also allows a firm to better absorb any cost increases forced on
it by powerful suppliers, because the cost leader has greater margins than its competitors. In fact, a
cost leader may be able to force its suppliers to keep prices low for them.
Entrants: The cost leadership strategy also discourages new entrants because the new entrant must be
willing to accept no better than average returns until they gain the experience and core competencies
required to approach the efficiency of the cost leader.
Substitutes: For substitutes to be used, they must not only perform a similar function but also be
cheaper than the cost leader's product. When faced with substitute products, the cost leader can reduce
its price.
4. Risks of a cost leadership strategy?
In a cost leadership strategy, the producer seeks to offer products with acceptable features to
customers at the lowest competitive price. One risk of a cost leadership strategy is that the firm's
investment in manufacturing equipment may be made obsolete through technological innovations by
competitors. Additionally, a firm with a cost leadership strategy may focus on cost reduction at the
expense of trying to understand customers' needs and/or competitive concerns. Finally, competitors
may be able to imitate a cost leader's competitive advantages in their own unique strategic actions.
5. Discuss how a differentiation strategy can allow a firm to earn above average returns in
spite of strong competitive forces. Address each of the five competitive forces.
Rivalry: Customers tend to be loyal purchasers of products that are differentiated in ways that are
meaningful to them. As their loyalty to a brand increases, customers' sensitivity to price increases is
reduced. The relationship between brand loyalty and price sensitivity insulates a firm from
competitive rivalry.
Buyers: The uniqueness of differentiated goods or services reduces customers' sensitivity to price
increases.
Suppliers: Because a firm using the differentiation strategy charges a premium price for its products,
suppliers must provide high-quality components, driving up the firm's costs. However, the high
margins the firm earns in these cases partially insulate it from the influence of suppliers because
higher supplier costs can be paid through these margins. Alternatively, because of buyers' relative
insensitivity to price increases, the differentiated firm might choose to pass the additional cost of
supplies on to customers by increasing the price of its unique product.
Entrants: Customer loyalty and the need to overcome the uniqueness of a differentiated product
generate substantial barriers for potential entrants. Entering an industry under these conditions
typically demands significant investments of resources and patience while seeking customers' loyalty.
Substitutes: Firms selling brand-name goods and services to loyal customers are positioned effectively
against product substitutes. In contrast, companies without brand loyalty face a higher probability of
their customers switching either to products that offer differentiated features that serve the same
function (particularly if the substitute has a lower price) or to products that offer more features and
perform more attractive functions.
=> The differentiation strategy is an integrated set of actions taken to produce goods or services (at an
acceptable cost) that customers perceive as being different in ways that are important to them. Product
innovation is critical to successful use of the differentiation strategy. If the firm has a thorough
understanding of what its target customers value, the relative importance they attach to the satisfaction
of different needs, and for what they are willing to pay a premium, the differentiation strategy can be
effective in helping it earn above-average returns.
6. Describe the risks of a differentiation strategy.
https://www.studyblue.com/notes/note/n/chapter-4-business-level-strategy/deck/7118779
https://quizlet.com/272280883/mgmt-4900-midterm-flash-cards/
The risks of a differentiation strategy include the fact that the price differential between the low-cost
producer and the differentiated firm’s product may be too high for the customer. The differentiated
products may exceed the customers’ needs. Additionally, differentiation may cease to provide value
for which customers are willing to pay. This can occur if rivals imitate the firm’s product and offer it
at a lower price. A third risk is that customer learning can narrow the customer’s perception of the
value of the firm’s differentiated product. If customers have positive experience with low-cost
products, they may decide the additional cost for the differentiated product is too high. Finally,
counterfeit products are a risk to a differentiation strategy if these products provide the same
differentiated features to customers at significantly reduced prices.
7. How do focused differentiation and focused cost-leadership strategies differ from their
non-focused counterparts?
Focus strategies target specific industry segments or niche rather than the entire market. The market
can be segmented into 1) a particular buyer group, 2) a different part of a product line, or 3) different
geographic areas. The firm using a focus strategy hopes to meet the needs of a particular target market
better than firms with a more broad-based approach. Or, they hope to meet needs of a market niche
that has been overlooked or neglected by broad-based rivals.
8. Describe the additional risks undertaken by firms pursuing a focus strategy.
Focus firms face three additional risks beyond the general risks of industry-wide strategies. First, a
competitor may be able to focus on a more narrowly defined competitive segment and “outfocus” the
focuser. Second, a firm competing on an industry-wide basis may decide the targeted market segment
is attractive and worthy of competitive pursuit. Finally, the needs of the firm’s customer group may
become more similar to the needs of industry-wide customers as a whole, thereby eliminating the
advantages of a focus strategy.
9. Describe the advantages of integrating cost leadership and differentiation strategies.
Customers have increasingly high expectations for products, wanting products that are both low-
priced and differentiated. So a number of firms are trying to simultaneously follow both a cost
leadership and a differentiation strategy. This requires the firm to perform the primary and support
activities required of both strategies, which is challenging. Successful integration of strategies allows
firms to adapt quickly to environmental changes, and learn new technologies. The firm gains more
skills which makes it more flexible. Evidence suggests that successful use of integrated strategies is
related to above-average returns. A number of firms such as Target Stores and European-based Zara
owe their success to the integrated cost leadership/differentiation strategy.
10. What are the risks of an integrated cost leadership/differentiation strategy?
Integrated strategies present risks that go beyond those that arise from the pursuit of any single
strategy by itself. Principal among these risks is that a firm becomes “stuck in the middle.” In such a
situation a firm fails to implement either the differentiation or the cost leadership strategy effectively.
The firm will not be able to earn above- average returns, and without favorable conditions, it will earn
below-average returns. Recent research suggests that firms using either cost leadership or
differentiation often outperform firms attempting to use a “hybrid” strategy (i.e., integrated cost
leadership/differentiation). This research suggests the risks associated with the integrated strategy.
Case Scenario 1: International Cow Packers. International Cow Packers (ICP) is a $12 billion
meat processor (slaughter, processing, and packing). Founded in 1943, ICP has grown to
become the largest beef and pork processor in the United States (revenues come 90% from beef
and 10% from pork) and also has a growing export market to Japan. The company follows a
focused cost-leadership strategy, delivering USDA-graded meats primarily to the institutional
(schools, prisons, hospitals) and supermarket channels. ICP's entire value chain is organized to
deliver volume product at the industry's lowest per-unit cost. Its supplier industries, primarily
cattle and swine feedlots, have relatively little power since prices for these raw materials are
determined in the commodity markets. While entry barriers to the industry are high due to high
minimum start-up costs, industry rivalry is extremely intense - primarily due to the fact that
three large companies (including ICP) control 80% of the market for processed meats. The
threat of substitutes is high with an increasing trend for consumers to favor poultry and other
non-beef proteins. Buyers are also powerful since supermarkets are relatively concentrated at a
regional level and end-consumers have ample choices. -(Refer to Case Scenario 1)
1. Is ICP's focused low-cost strategy appropriate for its industry? Why?
The best answers will begin by noting that ICP sells a commodity product, as evidenced by the fact
that there are only so many grades of USDA-certified beef and pork. Since the product is an
undifferentiated commodity, customers typically base their purchasing decisions on price alone.
2. What risks is ICP accepting by adopting its focused low-cost strategy?
The best answers will note that since ICP has aligned its entire value-chain with its low-cost strategy,
it has linked its own ups and downs to the ups and downs of the beef and pork industries. Thus, like
commodity prices, we can expect that ICP will do well when general demand for beef and pork is up,
and less well when such demand is down. A more nuanced answer may also point out that ICP’s
intense focus on costs may essentially drive out any opportunities for it to develop differentiation
advantages (other than offering the lowest cost product). If competitors are able to match ICP’s
efficiency as well as build other differentiation advantages (like brand management skills or forward
integration into value-added meat products like prepackaged meals), ICP may find itself at a
competitive disadvantage in the long run.
3. What can ICP do to decouple itself from the ups and downs of the pure commodity
markets? What specific actions might ICP undertake?
The best answers will begin by suggesting that ICP must retain its cost advantage while developing
differentiation advantages. At a general industry level, ICP can promote the consumption of beef and
pork to counter trends away from these meats. Specific to ICP, it can begin experimenting with value-
added products like prepackaged meals (frozen dinners, etc.). A related strategy would be the
development of organic products that do not fall within the USDA categories. Selling high-quality
beef and pork outside of the USDA categories would be another strategy as well. The theme across
students’ recommendations should be one of developing products that no longer have commodity-like
characteristics.
Case Scenario 2: Walt Disney Company. Walt Disney Company is famed for its creativity,
strong global brand, and uncanny ability to take service and experience businesses to higher
levels. In the early 1990s, then-CEO Michael Eisner looked to the fast-food industry as a way to
draw additional attention to the Disney presence outside of its theme parks - its retail chain was
highly successful and growing rapidly. A fast-food restaurant made sense from Eisner's
perspective since Disney's theme parks had already mastered rapid, high-volume food
preparation, and, despite somewhat undistinguished food and high prices (or perhaps because
of), all its in-park restaurants were extremely profitable. From this inspiration, Mickey's
Kitchen was launched. The first two locations were opened in California and in a suburb of
Chicago, adjacent to existing Disney stores. Menu items included healthy, child-oriented fare
like Jumbo Dumbo burgers and even a meatless Mickey Burger. Eisner thought that locating
each restaurant next to existing Disney stores was sure to increase foot traffic through both
venues. Less than two years later Disney closed down the California and Chicago stores and
shuttered further expansion plans. Eisner cited overwhelming competition from McDonalds
and general oversaturation in the fast-food industry as the primary reasons for closing down the
failing Mickey's Kitchen. -(Refer to Case Scenario 2)
1. Based on your own knowledge of Disney and the information provided in the scenario,
does Disney appear to create value in its businesses primarily through a cost-leadership
or through a differentiation strategy?
The best answers will begin by noting that Disney, via Mickey Mouse, is probably one of the world’s
most recognized brands. This unique asset complements a differentiation strategy well. Students may
further remark that while Disney may seek efficiencies in all of its operations, ticket prices for the
theme parks do not appear to be a particular bargain, and that Disney never seems to promote its
products based on their cost. This is illustrated by the point that the in-park restaurants charge high
prices.
=> Although the Mickey’s Kitchen failed, but Disney still appear to create value in its businesses
primarily through differentiation strategy. Disney’s strategy of differentiated more reflect the value of
health in upstream and downstream link, formed the unique brand value chain operation mode, and
realize the years of stable growth. After nearly 90 years of development, the company has become a
Disney film and television entertainment, with theme park, media network and merchandise of
business such as the world's most famous enterprise of the corpse, brand value, to $29.2 billion.
2. What resources and value-chain activities did Disney try to leverage through the
opening of Mickey's Kitchen?
It appears that Disney was hoping for a differentiation advantage through (1) the image of Mickey
Mouse, (2) its service management expertise, particularly in food service, and (3) locations next to its
already successful chain of retail outlets.
=> Disney is Walter Dean Disney Company's brand. Cartoon characters and fast food industry group
is the Disney in an operation brand portfolio strategy. Because Disney cartoon character is the Disney
starting period of the brand development, continue today. Tourists from cognitive perspective, the
movie cartoon characters such a brand into of fast food industry, can make visitors quickly built "fast
food kitchen-cartoon characters-Disney" brand association contact.
3. Why do you think that Mickey's Kitchen failed?
The best answers will begin with the observation that it is hard to imagine that Mickey’s Kitchen
could create the differentiated Disney experience and margins at fast-food prices. The discussion can
then be extended to note that Disney did deploy a set of resources that were valuable, rare, costly to
imitate, and non substitutable, but it did so in the fast-food industry where consumers make choices
based primarily on price. Thus, Disney’s particular resources generated differentiation advantages, but
not the needed cost advantages. It also can be pointed out that Disney’s theme park restaurants have
likely done well because guests of the park are a captive audience and have few food choice
alternatives unless they opt to leave its park or properties.
=> As we all know, Brand combination as a brand strategy important constituent, need to rely on
strong brand as a company, based in different levels, with the scientific method the brand portfolio. So
I think the reason of the Mickey’s Kitchen failed is the unscientific strategy. The unscientific parts are
due to overwhelming competition from McDonalds and Fast food industry is saturated. Competition
is the key to the success or failure of the enterprise. McDonald's corp., the most famous McDonald's
brand has more than 32000 home fast-food restaurants, distribution in 121 countries and regions. In
the world according to the taste of the local people to McDonald's meal adjusted. At present, in the
global fast food chain McDonald's field is the champion. In this respect, Mickey’s Kitchen is
completely without the advantages. Fast food industry market saturation is also the important reasons
that lead to the collapse. Because the fast-food industry is not Disney’s main industry, so the
resources of relatively that it takes into the resources of relatively is small.
Case Scenario 3: Abrahamson's Jewelers. Through its sole location in an affluent suburb of San
Francisco, Abrahamson's Jewelers has established a strong niche market in the upscale jewelry
store segment. Abrahamson's was founded in 1871 and is currently owned and operated by
John Wickersham, who bought the firm from its namesake founders in 1985. Wickersham
joined the firm as a trainee out of high school, completed his gemology training, and several
years later took ownership with the financial help of his parents. That debt has long been paid
off and business has thrived. When he first acquired the business, Abrahamson's offered a full
range of jewelry and gift items from watches to wedding sets to silverware to clocks. This broad
range of products was mirrored by a broad price range-$10,000 Rolex watches were sold next to
$50 Seiko watches. While some jewelry was custom designed and manufactured, most of the
products were "case ready," meaning they were sourced from large jewelry and silver
manufacturers from around the world. Over the last 15 years, Wickersham has narrowed the
company's product offering considerably to focus only on high-end watches like Rolex and
Piaget, custom jewelry, and estate jewelry. Wickersham stresses that this is an appropriate
focus for his business since each of the products lends itself to relationship selling, and price
rarely comes into the discussion. Despite the narrower offering moreover, Abrahamson's floor
space has doubled, and clients are intensely loyal to the good taste, design skills, and personal
service level provided by Mr. Wickersham. -(Refer to Case Scenario 3)
1. What generic business strategy best describes Abrahamson's? Why?
The best answers will observe that all the features of this case point to a focused differentiation
strategy. The company is focused both in terms of product offering and geography. Purchase
decisions are based primarily on a relationship with Mr. Wickersham and unique products, not on
price.
2. While Abrahamson's is doing well, Mr. Wickersham would like to grow his business
beyond the present location. He believes that growth may bring greater profitability, as well as
employment avenues for his only child, who will soon be finishing high school. What
recommendations do you have for Mr. Wickersham regarding his growth choices?
The objective here is to get students to see the limits to growth presented by Abrahamson’s current
strategy and key resources. This scenario also provides a nice opportunity to link a company’s
strategy and resource base with a key individual—in this instance, Mr. Wickersham. The best answers
will start by walking through a particular expansion plan and then noting how the company’s
resources do and do not support that plan. For instance, one obvious avenue is to open additional
locations. Such an avenue would likely leverage Wickersham’s contacts and expertise in sourcing raw
materials, as well as providing a greater market to exploit his representation and contracts with watch
firms like Rolex and Piaget. A second avenue would be to leverage Wickersham’s design skills to go
into the wholesale jewelry business. The risk underlying both of these growth avenues is that it may
spread Mr. Wickersham too thin: as the scenario clearly suggests, his personal knowledge and
relationships (and time) are central to Abrahamson’s current success.
3. Would you recommend that Mr. Wickersham embark on an Internet sales strategy for
his company?
The best answers will note that some aspect of the Internet may be valuable for Abrahamson’s, but
that his current resource base does not lend itself well to an Internet sales vehicle. Customers typically
expect that products sourced and sold online will be cheaper than through traditional retail channels,
even for high-end items like watches (for instance, have students do a Web search for Rolex watches).
Abrahamson’s is not poised to, nor does it seem inclined to, compete on price. In terms of customer
relations, however, Abrahamson’s could use some form of Internet presence to show its customers a
broader variety of products in its already narrow line. They could also perhaps see prior design work
to help them better imagine what a custom-designed piece might look like. Particularly for the estate
sales, Abrahamson’s could link its inventory to larger, reputable online estate sale houses—thus
giving its customers the benefit of local relationships with the power of the Internet’s worldwide
markets. Finally, use of the Internet for maintaining contact with existing customers would enhance
its relationship with and knowledge of them.

Quiz 5: Competitive Rivalry and Competitive Dynamics


https://wenku.baidu.com/view/fc888a8602d276a200292ec7.html
1. Define competitors, competitive rivalry, competitive behavior, and competitive
dynamics.
Competitors are firms competing in the same market, offering similar products, and targeting similar
customers.
Competitive rivalry is the ongoing set of competitive actions and competitive responses occurring
between competitors as they compete against each other for an advantageous market position.
For the individual firm, the set of competitive actions and responses it takes while engaged in
competitive rivalry is called competitive behavior.
Competitive dynamics is the set of actions and responses taken by all firms that are competitors
within a particular market
2. What is market commonality? What is resource similarity? How are these concepts
combined to identify the level of competition between two firms?
Market commonality is concerned with the number of markets with which the firm and a competitor
are jointly involved and the degree of importance of the individual markets to each. Each industry is
composed of several markets which are characterized by different customers, product segments, or
geographic areas, among other differentiators. In general, competitors agree about the different
characteristics of the individual markets that make up an industry. Most industries’ markets are
somewhat related in terms of technologies used or core competencies needed to develop a competitive
advantage. Firms may compete against one another in one market or in many, a situation termed
multimarket competition.
Resource similarity is the extent to which the firm’s tangible and intangible resources are comparable
to a competitor’s in terms of both type and amount. Firms with resource similarity are likely to have
similar strengths and weaknesses and to use similar strategies.
The combination of high or low market commonality and high or low resource similarity identifies
whether firms are competitors. Firms having both high market commonality and high resource
similarity are direct and mutually acknowledged competitors. If firms share few markets and have
little similarity in resources they are not direct and mutually acknowledged competitors.
3. Define awareness, motivation and ability in reference to competitive behavior.
Awareness, motivation and ability are the drivers of competitive behavior. They influence the firm’s
actions toward and responses to competitors. Awareness is the extent to which competitors recognize
the degree of their mutual interdependence that results from market commonality and resource
similarity. Awareness affects the extent to which the firm understands the consequences of its
competitive actions and responses.
Awareness is greatest when firms have highly similar resources. Motivation concerns the firm’s
incentive to take action against a competitor or to respond to a competitor’s attack. If the firm doesn’t
believe that attacking its competitors will improve its position, it will not act. If the firm does not
believe a competitor’s action will result in losses for it, it will not have motivation to respond. High
market commonality gives firms more motivation to attack and to respond to competitors’ actions
than when market commonality is low. Ability relates to each firm’s resources and the flexibility
these resources provide. When a firm faces a competitor with similar resources, careful study of a
possible attack is essential because a competitor with similar resources is likely to respond to
competitive attack. When the resources between two competitors are very dissimilar, the weaker firm
will delay in responding to an attack by the stronger firm.
4. Define competitive actions and responses and explain the two types of competitive
actions and responses.
This refers to strategic and tactical actions. Strategic actions take more time to implement, require
many specific resources, and are difficult to reverse. Tactical actions tend to be quicker to implement,
require fewer resources, and can be reversed more easily. Strategic actions tend to receive strategic
responses. Tactical actions tend to receive tactical responses, because they are easy to put into place.
Strategic actions elicit fewer total competitive responses than do tactical actions. Responses to
strategic actions will be slower than will responses to tactical actions because competitors need time
to observe whether the strategic action will be successful. But, if a competitor’s action threatens a
large number of a firm’s customers, the firm will react strongly regardless of whether the competitor’s
action is strategic or tactical.
5. What are the advantages and disadvantages of being a first mover, second mover, and
late mover?
First movers can gain market share, customer loyalty, and high revenues by being the first in the
market. But, first movers also take more risk because it is difficult to judge the returns the firm will
earn from product innovations. Moreover, if the first mover is successful, other firms will enter its
arena. First movers tend to have a significant amount of organizational slack to fund research and
development. Second movers imitate the first movers, after they have studied the first mover’s
successes and mistakes. Consequently second movers can develop more efficient processes and
technologies than first movers, which results in lower costs. Late movers react to the first and second
movers’ actions after a long delay. A late mover may be able to earn average returns if it has learned
how to create at least as much value for customers as the value created by the first and second movers.
In general, late movers are relatively ineffective.
6. What factors contribute to the likelihood of a response to a competitive action?
In general, a firm is more likely to respond to a competitive action if: (1) the action leads to better use
of the competitor’s capabilities to gain or produce stronger competitive advantage or to improve its
market position, (2) the action damages the firm’s ability to use its capabilities to create or maintain
an advantage, or (3) the firm’s market position becomes less defensible. In addition, a firm is more
likely to respond to a competitor’s tactical action, rather than to a competitor’s strategic action.
Strategic actions involve a significant commitment of resources and are difficult to implement and
reverse, as well as requiring time to put into place. In contrast, tactical actions can be implemented
quickly and are quickly reversed, and are relatively less costly than strategic actions. A firm is also
more likely to respond to a competitor’s action when the competitor is the market leader - a firm that
has the reputation for above-average returns. Successful actions by competitors are likely to be
quickly imitated, even if not initiated by a market leader. Actions by price predators are usually not
responded to, nor are actions by firms with reputations for risky, complex, and unpredictable
behavior. Finally, competitors with high market dependence are likely to respond strongly to attacks
threatening their market position.
7. Define slow-cycle, fast-cycle and standard cycle markets.
In slow-cycle markets, the firm’s competitive advantage is shielded from imitation for long periods of
time and imitation is costly. Competitive advantages are sustainable in slow-cycle markets. Successful
firms in slow-cycle markets have difficult-to-understand and costly-to-imitate advantages resulting
from unique historical conditions, causal ambiguity and/or social complexity. These conditions can
include copyrights, patents, and ownership of an information resource. Firms in slow-cycle markets
focus on protecting their competitive advantages and exploiting them as long as possible. In fast-cycle
markets, imitation happens quickly. Competitive advantages are not sustainable. Reverse engineering
and quick technology diffusion facilitate rapid imitation. In fast-cycle markets, innovation is critical
and firms avoid “loyalty” to any product. Firms must focus on rapidly and continuously developing
new competitive advantages, because prices fall quickly and firms need to profit rapidly from
innovations, and move on to the next product. Fast-cycle markets are volatile and the pace of
innovation is frenzied. In standard-cycle markets, the firm’s competitive advantages are moderately
shielded from imitation and imitation is moderately costly. Competitive advantages are partially
sustainable if the firm can continuously upgrade the quality of its capabilities making its competitive
advantage dynamic. Typically, these markets have large firms seeking high market share, striving for
customer brand loyalty, and controlling their operations to give customers consistent experiences.
Economies of scale are necessary for survival. Competition for market share is intense and is often
based on incremental innovation in a product rather than radical innovation.
Case Scenario 1: Romulac, Inc. Romulac Inc. (RI), a subsidiary of a large successful
manufacturing conglomerate, supplies a key component in the assembly of residential cooling
systems (air conditioning units, etc.). There has been tremendous consolidation in RI's industry,
to the point where only five suppliers of this particular component account for nearly 90% of
U.S. industry sales. Paralleling this trend, its customers - comprised of makers of branded
residential air conditioning units like Carrier and Trane - have seen similar levels of
consolidation in their own industry. Half of these firms produce all their components in-house,
while the balance purchases them from specialized component manufacturers like RI. RI's
business is extremely capital intensive, and their 40% share of the market allows them to also be
the most profitable domestic player. Strong competitors exist in Europe and Asia. Although like
RI, these foreign players' strongholds are their home regions, with negligible presence outside of
the region. Some of the larger Asian manufacturers have signaled an interest in more
aggressively pursuing the lucrative U.S. market. RI is presently considering a $400 million
dollar investment in a new plant, which will create a component that is much quieter, more
efficient, and is likely to satisfy future regulatory standards. While the core technology for the
new component is very old, RI's engineering and design skills have allowed them to retain their
low cost-advantage, even though the component will represent a significant improvement over
products currently provided by its competition. -(Refer to Case Scenario 1)
1. Develop an argument as to why RI should try to be a first-mover with this new
technology.
The best answers will begin by suggesting that RI move quickly to retain its dominant market share,
particularly since the technology itself is not new, and competitors may easily develop their own
efficient designs. A more subtle argument is that RI has an opportunity to set a new industry
standard,and as the leader, may likely gain even greater market share.
2. Develop an argument as to why RI should hold back and be a second mover with the
new technology.
The best answers will observe that RI cannot predict with certainty that its new technology will be
cheaper, or as cheap to produce. Thus, a primary risk is that RI invests in the plant, the industry moves
to the new technology, but the technology is actually more costly to produce - in this way, RI may
cannibalize its existing low-cost position with a higher-cost one. By waiting, RI can learn from its
competitors’ mistakes. A secondary risk is that competitors will learn from RI’s initial mistakes, and
be able to offer the new technology for considerably lower cost. The nightmare scenario here is that
the industry moves to the new technology, RI has a higher cost position, and overseas competitors
steal domestic market using the new technology, which they have learned to manufacture at a lower
cost. Thus, RI could avoid this latter risk by again waiting out the competition.
3. As one of RI's direct competitors, how would you try to predict what it will do with
regard to the new technology?
The best responses can begin by pointing to three main characteristics that are likely to heavily
influence RI’s choices. The ways these factors bode in favor and against the move should be
discussed. First, RI is both large and a dominant player in this market. Second, RI is very profitable
and such profitability is a direct consequence of its large market share. Finally, RI is the subsidiary of
a large successful conglomerate - students would want to point out that the new technology will
require a large corporate commitment ($400 million). A related issue is whether the corporate parent
considers itself to be primarily a first or second mover in its competitive interactions.
Case Scenario 2: Plasco. Plasco is a $3 billion U.S.-based manufacturer of flexible plastic
products like trash cans, reheatable and freezable food containers, and a broad range of other
plastic storage containers designed for home and office use. Historically, Plasco has been the
category killer for most of its products and has devoted tremendous resources to new product
development on an ongoing basis - this research intensity has allowed the company to release,
on average, a new product every day over the past five years. Despite its past strength and high
brand awareness, Plasco's profitability has been eroded by dramatic increases in the cost of
plastic resin, the primary input into its plastic products. Moreover, the retail channel has
experienced rapid consolidation resulting in a shift in the balance of power from branded
manufacturers like Plasco, to strong retailers like Wal-Mart, who in turn have been unwilling to
help Plasco absorb the higher resin costs. Enhancing Wal-Mart's power is the fact that it can
always turn to alternative high-volume sources of consumer plastic products like Sterlite.
Further hampering Plasco's recovery is the emergence of feisty little foreign competitors like
Zig Industries, a $250 million Israeli firm that has begun to take part of Plasco's market share
in plastic toolboxes. Ironically, Plasco was the first company to offer plastic toolboxes some 20
years ago. This innovation changed the market dramatically and Plasco's first mover strategy
rewarded it with a rapidly growing new segment and a dominant market position. Today,
Plasco's toolboxes are viewed as rather boring, while Zig's products are ingeniously designed to
catch the customer's eye in the aisle (better merchandising the product) and capture their
interest (and pocketbook) with many new and novel features. Zig is also able to provide this new
line of toolboxes at between 10% to 15% less than Plasco. -(Refer to Case Scenario 2)
1. Is Walmart a competitor or a customer of Plasco?
The best answers will start by summarizing that Wal-Mart is both a customer and a competitor. It is a
customer in the sense that it is a primary outlet for Plasco’s products. Wal-Mart is a competitor from
the standpoint that Walmart has control over Plasco’s profitability and, in a sense, is competing for a
portion of the profit pool in Plasco’s industry as well.
2. Is the toolbox business a slow-, standard-, or fast-cycle business?
The best answers will suggest that the cycle characteristics of the toolbox market appear to have
shifted over time. Before Plasco entered the business, metal toolboxes were the norm and this was
likely to be characterized as standard to slow in terms of its cycle speed. The metal toolbox market
was probably an oligopolistic one, dominated by a few profitable players. With the entrance of Plasco
and its plastic toolboxes, the cycle speed among metal toolbox manufacturers increased, where they
no longer dominated the industry. Plasco’s innovative product plus its unique capacity (at the time) to
produce a durable plastic toolbox probably turned its segment into a slow-cycle market (with Plasco
enjoying a near monopoly position), while the metal toolbox took on standard- to fast-cycle market
characteristics. With the new entrance of Zig into plastic toolboxes, this segment is now likely to be
characterized as standard to fast cycle - plastic technologies aren’t proprietary and designs are readily
copied by competitors.
3. How can a small player like Zig be such a successful competitor against a large,
established firm like Plasco?
The value of this question is that it forces students to consider how changes in both a focal industry
and its upstream industry may affect competition. The best answers can begin by noting that the
market for plastic toolboxes is probably pretty large (tools, cosmetics, fishing gear, toys, etc),
especially on a global basis. Couple this observation with the fact that there are a number of mega-
retailers who would find this to be a necessary product to stock on their shelves. Thus, this market
segment is nearly an industry in and of itself and a small focused player could gain economies of scale
in manufacturing as well as distribution and marketing to the staple, volume retailers like Wal-Mart,
Home Depot, and Carrefour. Finally, Plasco was apparently treating the plastic toolbox market as a
stable one, leaving it less likely to invest much of any additional resources into further innovation.
This creates a window of opportunity for a nimble, aggressive, focused and talented new entrant like
Zig.

Quiz 6: Corporate-Level Strategy


https://www.studyblue.com/notes/note/n/chapter-6-corporate-level-
strategy/deck/7118898#:~:text=The%20five%20categories%20of%20businesses
%20determined%20by%20level%20of%20diversification,per%20cent%20of%20revenue
%20from
https://quizlet.com/84790104/essay-questions-for-chapter-6-flash-cards/
1. Differentiate between corporate-level and business-level strategies and give examples of
each.
A business-level strategy determines how a firm will compete in a single industry or product market.
When a firm diversifies beyond a single industry it uses a corporate- level strategy. A diversified
company has two levels of strategy: business-level and corporate-level. Each business unit has a
business level strategy. The corporate strategy is concerned with: 1) what businesses the firm should
be in and 2) how the corporate office should manage the group of businesses. The top management of
diversified companies views the firm's businesses as a portfolio of core competencies that will
generate above-average returns by creating value. An example of a business-level strategy would be
whether the firm targets the mass market and competes on price, or whether it competes on the basis
of uniqueness. An example of a corporate-level strategy would be whether the firm should sell off a
poorly performing subsidiary.
2. What are the five categories of businesses based on level of diversification?
The five categories of businesses determined by level of diversification are as follows: (1) Single
business (more than 95 percent of revenues from a single business), (2) Dominant business (between
70 percent and 95 percent of revenue from a single business), (3) Related constrained (a diversified
organization earning less than 70 percent of revenue from the dominant business, and all the
component businesses share product, technological, and distribution linkages), (4) Related linked (a
diversified organization earning less than 70 percent of revenues from the dominant business with
only limited links among the component businesses), and (5) Unrelated (diversified organizations
earning less than 70 percent of revenues from the dominant business with no common links among
the businesses).
3. Describe the primary reasons a firm pursues increased diversification.
Firms typically diversify to increase the firm's value by improving its overall performance. Value-
creating diversification occurs through related or unrelated diversification when the strategy allows
the company's business units to increase revenues or reduce costs while implementing business level
strategies. Alternatively, a firm may diversify to gain market power over competitors. Value-neutral
diversification may occur in response to governmental policies, firm performance problems, or
uncertainties about future cash flows. Finally, managers may have selfish motives to diversify, such
as increased compensation or personal reduced employment risk. These selfish motivations may
actually erode the firm's competitiveness, and can be value-reducing diversifications.
4. Describe how diversified firms can use activity sharing and transfer of core
competencies to create value.
In related diversification, a firm seeks to exploit economies of scope between its business units.
Economies of scope are cost savings created by transferring some of its capabilities and competencies
developed in another business to a new business. Firms create value through economies of scope two
ways: the sharing of activities (operational relatedness) and the transferring of core competencies
(corporate relatedness). Both primary and support activities may be shared, including marketing and
production. This activity sharing can result in cost reductions and improve financial returns. The
sharing of core competencies allows the firm to create value two ways: 1) it eliminates the need for
the second unit to allocate resources to develop the competence, and 2) transferring intangible
resources internally makes it hard for competitors to understand and to imitate the resource.
5. What are the two ways that an unrelated diversification strategy can create value?
Unrelated diversification can create value through two types of financial economies (cost savings). 1)
Unrelated diversified firms can more efficiently allocate capital among the component businesses than
can the external financial market. This is possible because the corporate-level management has more
complete information about the performance of the component businesses and it can also discipline
under- performing management teams. 2) Unrelated diversified firms can also create value by
purchasing other businesses at low prices, restructuring them, and reselling them at a higher price.
This practice is most successful with mature, low-technology businesses, rather than high-technology
or service businesses, which are more dependent on employees who may leave.
6. What is the effect of a firm's low performance on the pursuit of diversification?
High corporate performance eliminates the need for diversification. Some research shows that low
returns are related to greater levels of diversification. Firms plagued by poor performance often
diversify in an effort to become more profitable. But, continued poor performance following
diversification may slow the pace of diversification and may lead to divestitures and a focus on the
core business. In addition, firms that are more broadly diversified compared to their competitors may
have lower overall performance. Figure 6.3 shows that the related constrained diversification strategy
is the highest performing strategy. So poor performing firms that intend to diversify should look at
purchasing businesses that would be suitable for this strategy rather than moving into unrelated
diversification or retaining a dominant business strategy.
7. What are the managerial motives to diversify?
A top-level manager may be motivated to pursue diversification because diversification leads to
greater job security for executives. In general, greater amounts of diversification reduce managerial
risk because if a particular business fails, the top executive remains employed by the corporation. In
addition, diversification increases firm size, and firm size has a direct effect on executive
compensation. Moreover, managing a highly diversified firm is more difficult; thus, managerial
compensation is generally higher in such a firm. Consequently, executives may have selfish motives
to diversify the company in ways which may actually reduce corporate competitiveness.
Case Scenario 1: Syco. Syco is a diversified company that has six primary lines of business. Fifty
percent of its revenues and 18 percent of its profits come from retailing. Most of its retail outlets
are discount department stores that serve as anchor tenants for large suburban shopping malls.
The remaining businesses are broken out as follows: Insurance accounts for 30 percent of
revenues and 50 percent of profits; consumer credit card operations are 6 percent of sales and
17 percent of profits; 5 percent of revenues and 6 percent of profits come from its stock
brokerage business; commercial and residential real estate operations generate 4 percent of
sales and 8 percent of profits; finally, 5 percent of revenues and 1 percent of profits come from
its online portal business. The company's management states that all these businesses are
essential to its competitive future. -(Refer to Case Scenario 1)
1. Why might there be so much variability among the proportion of sales versus
profitability contributed by each of the businesses? Does this mean that Syco is more successful
in its insurance business than in its retail business?
The best answers to this question will start out by noting that industries vary in their profit structures.
That is the margins in retailing are typically very low, while those in insurance are relatively much
higher. Beyond this, the statement tells us nothing about how well Syco is actually doing in each
business, since you would need to compare business-level performance against that of competitors on
a business-by-business basis. [Note to the instructor: The above scenario is based loosely on Sears in
the mid-1980s. At that time its lines of business were Sears Department Store, Allstate Insurance,
Discover Card, Dean Witter Stock Brokerage, Coldwell Banker Real Estate Brokers, and Prodigy
Online. Sears eventually divested all but its department stores and, at one time, was near bankruptcy
under the weight of its diverse operations and failing retail business (under-maintained mall properties
became a core rigidity).]

2. Part 1: (Refer to Case Scenario 1) Develop a logical argument that would lead you to
describe Syco's diversification type as related linked and another logical argument that Syco's
diversification type is related constrained. For both the related linked and for the related
constrained arguments, what product, technological, or distribution activities might link these
businesses together? Part 2: Would you describe either of the logical arguments you developed
in response to Part 1 as a good corporate strategy?
Part 1: The purpose of this exercise is to give students an opportunity to view the diversified firm
from the perspective of the top management team. The best answers will start with related linked
strategies and note that superficial linkages can be provided by global brandings (i.e., all businesses
under one name), one-stop shopping (i.e., all businesses under one roof), and shared accounting
systems (i.e., centralized accounting, cash allocation, and planning). The best answers to the related
constrained questions will offer more complex linkages beyond those noted above like shared
customer lists, active cross-selling, and rotation of key personnel. Because the businesses are so
different, however, the students should begin to see that more complex linkages may be difficult to
achieve.
Part 2: Based on the challenges of developing answers for Part 1, this question forces students to stand
back and criticize the strategies that they created. Since there are many possible approaches to Part 1,
it should become clear to students that each of these competing arguments has significant drawbacks.
Ultimately, students should reach the point where they may agree that offering a bundle of services is
desirable, although there is no reason that Syco has to own all of these offerings. This discussion also
provides a nice way to foreshadow the role and importance of strategic alliances which are covered in
Chapter 9.

Case Scenario 2: Jewell Company. Jewell Company (JC) is a $2 billion diversified manufacturer
and marketer of simple household items, cookware, and hardware. In the early 1950s, JC's
business consisted solely of manufactured curtain rods that were sold through hardware stores
and retailers like Sears. Since the 1960s however, the company has diversified extensively
through acquisition into such businesses as paintbrushes, writing pens, pots and pans, and
hairbrushes. Over 90 percent of its growth can be attributed to these many small acquisitions,
whose performance it improved tremendously through aggressive restructuring and its
corporate emphasis on cost-cutting and cost controls. While JC's sixteen different lines of
business may appear quite different, they all share the common characteristics of being staple
manufactured items and sold primarily through volume retail channels like Wal-Mart, Target,
and Kmart. Because JC operates each line of business autonomously (separate manufacturing,
R&D, and selling responsibilities for each line), it is perhaps best described as pursuing a
related linked diversification strategy. The common linkages are both internal (accounting
systems, product merchandising skills, and acquisition competency) and external (distribution
channel of volume retailers). JC is presently contemplating the acquisition of Plastico, a $3
billion U.S.-based manufacturer of flexible plastic products like trash cans, reheatable and
freezable food containers, and a broad range of other plastic storage containers designed for
home and office use. While Plastico has been highly innovative (over 80% of its growth has
come from internal new product development), it has had difficulty controlling costs and is
losing ground against powerful customers like Wal-Mart. JC believes that the market power it
wields with retailers like Wal-Mart will help it turn Plastico's prospects around. -(Refer to Case
Scenario 2) https://wenku.baidu.com/view/2377e989f90f76c660371a19.html
1. How might JC's related diversification strategy result in economies of scope and market
power?
For the question of efficiencies, the best answers will observe that:
(1) JC is able to leverage its accounting systems, product merchandising skills, acquisition
competency, and distribution channel of volume retailers across all its businesses (economies of
scope).
(2) These scope economies, in turn, provide JC the opportunity to build scale economies in these
resources and value chain activities.
(3) The market power dimension comes into play in terms of the scope economies it realizes within
the channel-for instance, JC's multi-product portfolio is likely to give it more power with regard to
Wal-Mart than would be possessed by a single product firm like Plastico.
=> Since the 1960s however, the company has diversified extensively through acquisition into such
businesses as paintbrushes, writing pens, pots and pans, and hairbrushes. Over 90 percent of its
growth can be attributed to these many small acquisitions, whose performance it improved
tremendously through aggressive restructuring and its corporate emphasis on cost-cutting and cost
controls. It is perhaps best described as pursuing a related linked diversification strategy.
2. Why would the acquisition of Plastico be good for JC?
The best answers will note that (1) JC has likely created some market power with respect to the large
retailers so another staple consumer product makes sense. (2) JC can leverage its existing market
presence, selling contacts, distribution system, and merchandising skills for plastic consumer products
as well. (3) And since Plastico is having trouble controlling its costs, JC can bring to it more
sophisticated financial management skills and accounting systems.
=> Plastico, a $3 billion U.S.-based manufacturer of flexible plastic products like trash cans,
reheatable and freezable food containers, and a broad range of other plastic storage containers
designed for home and office use. The acquisition of Plastico can help JC enlarge industrial
diversification. JC was already able to different business through the retail channel together, expand
their marketing advantage. From this, JC believes that the market power it wields with retailers like
Wal-Mart will help it turn Plastico’s prospects around.
3. What difficulties might you expect JC to encounter related to its acquisition of Plastico?
The best answers will begin by quickly noting that (1) Plastico is much bigger than JC, which itself is
likely to create problems (a nice lead-in to chapter 7, Acquisition and Restructuring). (2) And while
JC has grown through acquisition, it has primarily made small ones, which suggests that the firm may
be ill-prepared for the enormous challenge of taking control of such a large firm. (3) And even
though Plastico appears to be in trouble controlling costs, a more subtle observation is that JC's cost
control emphasis may be too extreme for Plastico's historic competency in innovation and, ultimately,
undermine this valuable resource.
=> The main difficulty is other takeover of the competition. Because of other investors to join, the
acquisition of cost will increase. Meanwhile, the JC sales in this aspect of the advantage may be
affected by the impact of other companies. Next is the acquisition of the timing of. The company is
the first step of options to buy the right time. In fact, for the company to make acquisitions in goal, to
their own have a clear, reasonable assessment of the target company has a clear orientation. At the
same time, also analysis the macroeconomic environment, legal environment and social environment,
etc. On the other hand, the purchase risk analysis is also a problem. Company purchase is high risk
management, the purchase risk very complicated and widely, the company should be treated with
caution, try to avoid risk, the risk in the acquisition and eliminate every aspect of, in order to purchase
success. In a word, in a takeover, to buy a company mainly face the risk: the market risk and
operational risk, the risk and financing risk, lose legal risk, and integrate the risk, etc.
Quiz 7: Merger and Acquisition Strategies
https://quizlet.com/61443725/busa-ch-7-flash-cards/
https://quizlet.com/85342340/strategic-management-ch-7-essay-flash-cards/
1. How have changing conditions in the external environment influenced the type of M &
A activity firms pursue?
During the recent financial crisis, tightening credit markets made it more difficult for firms to
complete mega deals (those costing $10 billion or more). As a result, many acquirers focused on
smaller targets with a niche focus that complemented their existing businesses. In addition, the
relatively weak U.S. dollar increased the interest of firms from other nations to acquire U.S.
companies.
2. How difficult is it for merger and acquisition strategies to create value and which firms
benefit the most from M & A activity?
Evidence suggests that using merger and acquisition strategies to create value is challenging. This is
particularly true for acquiring firms in that some research results indicate that shareholders of
acquired firms often earn above-average returns from acquisitions while shareholders of acquiring
firms typically earn returns that are close to zero. In addition, in approximately two-thirds of all
acquisitions, the acquiring firm's stock price falls immediately after the intended transaction is
announced. This negative response reflects investor's skepticism about the likelihood that the acquirer
will be able to achieve the synergies required to justify the premium.
3. Identify and explain the seven reasons firms engage in an acquisition strategy.
(1) Increased market power. Market power allows a firm to sell its goods or
services above competitive levels or when the costs of its primary or support activities are below
those of its competitors. Market power is derived from the size of the firm and the firm's resources
and capabilities to compete in the marketplace. Firms use horizontal, vertical, and related acquisitions
to increase their size and market power.
(2) Overcoming entry barriers. Firms can gain immediate access to a market by purchasing a firm
with an established product that has consumer loyalty. Acquiring firms can also overcome economies
of scale entry barriers through buying a firm that has already successfully achieved economies of
scale. In addition, acquisitions can often overcome barriers to entry into international markets.
(3) Reducing the cost of new product development and increasing speed to market. Developing new
products and ventures internally can be very costly and time consuming without any guarantee of
success. Acquiring firms with products new to the acquiring firm avoids the risk and cost of internal
innovation. In addition, acquisitions provide more predictable returns on investments than internal
new product development. Acquisitions are a much quicker path than internal development to enter a
new market, and they are a means of gaining new capabilities for the acquiring firm.
(4) Lower risk compared to developing new products internally. Acquisitions are a means to avoid
internal ventures (and R&D investments), which many managers perceive to be highly risky.
However, substituting acquisitions for innovation may leave the acquiring firm without the skills to
innovate internally.
(5) Increased diversification. Firms can diversify their portfolio of business through acquiring other
firms. It is easier and quicker to buy firms with different product lines than to develop new product
lines independently.
(6) Reshaping the firm's competitive scope. Firms can move more easily into new markets as a way to
decrease their dependence on a market or product line that has high levels of competition.
(7) Learning and developing new capabilities. By gaining access to new knowledge, acquisitions can
help companies gain capabilities and technologies they do not possess. Acquisitions can reduce inertia
and help a firm remain agile.
4. Describe the seven problems in achieving a successful acquisition.
(1) Integration difficulties. It may be difficult to effectively integrate the acquiring and acquired firms
due to differences in corporate culture, financial and control systems, management styles,
and status of executives in the combined firms. Turnover of key personnel from the acquired firm is
particularly negative.
(2) Inadequate evaluation of target. Due diligence assesses where, when, and how management can
drive real performance gains through an acquisition. Acquirers that fail to perform effective due
diligence are likely to pay too much for the target firm.
(3) Large or extraordinary debt. Acquiring firms frequently incur high debt to finance the acquisition.
High debt may prevent the investment in activities such as research and development, training of
employees, and marketing that are required for long-term success. High debt also increases the risk of
bankruptcy and can lead to downgrading of the firm's credit rating.
(4) Inability to achieve synergy. Private synergy occurs when the acquiring and target firms' assets are
complementary in unique ways, making this synergy difficult for rivals to understand and imitate.
Private synergy is difficult to create. Transaction costs are incurred when firms seek private synergy
through acquisitions. Direct transaction costs include legal fees and investment banker charges.
Indirect transaction costs include managerial time to evaluate target firms, time to complete
negotiations, and the loss of key managers and employees following an acquisition. Firms often
underestimate the indirect transaction costs of an acquisition.
(5) Too much diversification. A high level of diversification can have a negative effect on
the firm's long-term performance. For example, the scope created by diversification often causes
managers to rely on financial controls rather than strategic controls because the managers cannot
completely understand the business units' objectives and strategies. The focus on financial controls
creates a short-term outlook among managers and they forego long-term investments. Additionally,
acquisitions can become a substitute for innovation, which can be negative in the long run.
(6) Managers overly focused on acquisitions. Firms that become heavily involved in acquisition
activity often create an internal environment in which managers devote increasing amounts of their
time and energy to analyzing and completing additional acquisitions. This detracts from other
important activities, such as identifying and taking advantage of other opportunities and interacting
with importance external stakeholders. Moreover, during an acquisition, the managers of the target
firm are hesitant to make decisions with long-term consequences until the negotiations are completed.
(7) Growing too large. Acquisitions may lead to a combined firm that is too large, requiring extensive
use of bureaucratic controls. This leads to rigidity and lack of innovation, and can negatively affect
performance. Very large size may exceed the efficiencies gained from economies of scale and the
benefits of the additional market power that comes with size.
5. Describe how an acquisition program can result in managerial time and energy
absorption.
Typically, a substantial amount of managerial time and energy is required for acquisition strategies if
they are to contribute to a firm's strategic competitiveness. Activities with which managers become
involved include those of searching for viable acquisition candidates, completing effective due
diligence processes, preparing for negotiations and managing the integration process after the
acquisition is completed. Company experience shows that participating in and overseeing the
acquisition activities can divert managerial attention from other matters that are linked with long-
term competitive success (e.g., identifying and acting on other opportunities, interacting effectively
with external stakeholders).
6. What are the attributes of a successful acquisition program?
(1) The acquired firm has assets or resources that are complementary to the acquiring firm's core
business.
(2) The acquisition is friendly.
(3) The acquiring firm conducts effective due diligence to select target firms and evaluates the target
firm's health (financial, cultural, and human resources).
(4) The acquiring firm has financial slack.
(5) The merged firm maintains low to moderate debt.
(6) The acquiring firm has sustained and consistent emphasis on R&D and innovation.
(7) The acquiring firm manages change well and is flexible and adaptable.
7. What is restructuring and what are its common forms?
Restructuring refers to changes in a firm's portfolio of businesses and/or financial structure. There are
three general forms of restructuring:
(1) Downsizing involves reducing the number of employees, which may include decreasing the
number of operating units.
(2) Downscoping entails divesting, spinning-off, or eliminating businesses that are not related to the
core business. It allows the firm to focus on its core business.
(3) A leveraged buyout occurs when a party (managers, employees, or an external party) buys all the
assets of a (publicly traded) business, takes it private, and finances the buyout with debt. Once the
transaction is complete, the company's stock is no longer publicly traded.
8. What are the differences between downscoping and downsizing and why are each used?
Downsizing is a reduction in the number of employees. It may or may not change the composition of
businesses in the company's portfolio. In contrast, the goal of downscoping is to reduce the firm's
level of diversification. Downsizing is often used when the acquiring form paid too high a premium to
acquire the target firm or where the acquisition created a situation in which the newly formed form
had duplicate organizational functions such as sales or manufacturing. Downscoping is
accomplished by divesting unrelated businesses. Downscoping is used to make the firm less
diversified and allow its top-level managers to focus on a few core businesses. A firm that
downscopes often also downsizes at the same time.
9. What is an LBO and what have been the results of such activities?
Leveraged buyouts (LBOs) are a restructuring strategy. Through a leveraged buyout, a (publicly
traded) firm is purchased so that it can be taken private. In this manner,
the company's stock is no longer publicly traded. LBOs usually are financed largely through debt, and
the new owners usually sell off a number of assets. There are three types of LBOs: management
buyouts (MBOs), employee buyouts (EBOs), and whole-firm buyouts. Because they provide
managerial incentives, MBOs have been the most successful of the three leveraged buyout types.
MBOs tend to result in downscoping, an increased strategic focus, and improved performance.
10. What are the results of the three forms of restructuring?
Downsizing usually does not lead to higher firm performance. The stock markets tend to evaluate
downsizing negatively, as investors assume downsizing is a result of problems within the firm. In
addition, the laid-off employees represent a significant loss of knowledge to the firm, making it less
competitive. The main positive outcome of downsizing is accidental, since many laid-off employees
become entrepreneurs, starting up new businesses. In contrast, downscoping generally improves firm
performance through reducing debt costs and concentrating on the firm's core businesses. LBOs have
mixed outcomes. The resulting large debt increases the financial risk and may end in bankruptcy. The
managers of the bought-out firm often have a short-term and risk-averse focus because the acquiring
firm intends to sell it within 5 to 8 years. This prevents investment in R&D and other actions that
would improve the firm's core competence. But, if the firms have an entrepreneurial mindset, buyouts
can lead to greater innovation if the debt load is not too large.

Case Scenario 1: Syco Inc. (SI). Syco, Inc. (SI) was founded the late 1800s and grew through
acquisition from being primarily a large discount retailer into a highly diversified firm. Beyond
retailing (still SI's dominant business), by the middle of the 1990s its lines of business included
significant market positions in insurance, consumer credit cards, stock brokerage, commercial
and residential real estate brokerage, and an online Internet portal. Each of the non-retail
businesses was average in its relative industry performance. Consistent with the decentralized
structure at SI and arms-length corporate oversight, each of these businesses was also rapidly
developing their own unique brands and customer following. However, within a short period of
time it became apparent that the retail business was failing. SI's vast mall-based department
store holdings were suffering from deferred maintenance and merchandising that did not
appear to be popular with its once large consumer base. At the same time, highly efficient and
focused low-cost competitors like Wal-Mart were beginning to take significant market share
from SI. On the verge of bankruptcy by early 2000, SI's management chose to sell off its
insurance, real estate and stock brokerage units; it also spun off its credit card and portal
businesses in separate public offerings. -(Refer to Case Scenario 1)
1. Why do you suppose SI entered the non-retail businesses through acquisition? Is this a
cheaper route than starting up these businesses from scratch?
The best answers may begin by noting that SI had no real prior experience in these non-retail
businesses so they needed to either buy the relevant operations and skills or start them up from
scratch. Absent such experience it is considerably more expedient to enter these businesses through
acquisition, since they are likely to be able to acquire both the business and an experienced
management team. The second question gets to the fact that SI would also likely have to pay a
premium for the acquired firms since it brought no industry-specific knowledge to the bargaining
table.
=> I suppose SI entered the non-retail businesses through acquisition is a cheaper route than starting
up these businesses from scratch. The most direct benefits is, it can use the target company's
resources, including human, material, and relationship between all resources, to adjust his capital
structure, enterprise structure, so as to improve performance. And can also be bought with the
company had the reputation of the image, brand, to strengthen our market position, increase
operational profits. At the same time, SI can be greatly entered the non-retail businesses and have
time to market through the acquisition. Including the preparation stage, into the stage, the time
required to expand stage. On the other hand, through to the management of the purchase to also can
make SI fast have the normal operation of the function. So I think acquisition is a cheaper route than
starting up these businesses from scratch.
2. Part 1: (Refer to Case Scenario 1) Why do you suppose that SI sold off or spun-off its non-
retail businesses? Part 2: What should SI do after selling off the non-retail businesses?
Part 1: The best answers will note that SI was probably in too many and too many different
businesses. Each of these businesses had to compete in their respective industries while at the same
time dealing with SI's corporate ownership. By getting out of the non-retail business SI is able to get
back to its roots in retail. While of course speculative, students can debate whether or not SI chose the
right business to focus its future on. From a resource-based perspective, retail had the strongest
history, which would likely give SI the richest and most defensible set of valuable, rare, and costly to
imitate resources in the retailing business.
Part 2: This is a natural follow-on to Part 1 above. Students could begin this answer by suggesting that
SI's diversification strategy diverted its attention from the needs of its core retailing business. Future
efforts should be directed toward turning the retailing business around and aggressively trying to
outmaneuver emerging and existing retailing competitors. The instructor can use this dialogue to point
out that after establishing a strong industry position, SI probably viewed its retailing business as stable
and unthreatened, and thus used it as means of financing its broad diversification efforts. In contrast,
emerging companies like Wal-Mart viewed retailing as a growth vehicle and developed novel and
lower cost structures which eventually undermined the advantages established earlier by SI.

=> From the paragraph, we can know each of the non-retail businesses of SI was average in its
relative industry performance. SI’s vast mall-based department store holdings were suffering from
deferred maintenance and merchandising that did not appear to be popular with its once large
consumer base. So its retail business basically is not developed. At the same time, in the retail market,
high efficiency and low cost on the competition like Wal-Mart enterprise have more advantage. This
is because like Wal-Mart enterprise focusing only on retail market. And SI would because of other
retail industry distracted company's energy. When SI sold off or spun-off its non-retail businesses, it
could focus on company's all resources and take the retail market share preempted back.

Case Scenario 2: Raptec Raptec was incorporated in 1991 and went public on the Nasdaq Stock
Market in 1996. Raptec's strategy is to become the global leader in innovative storage solutions.
Raptec is an S&P 500 and a Nasdaq Stock Market 100 member. The company's hardware and
software solutions for eBusiness and Internet applications move, manage, and protect critical
data and digital content. Raptec operates in three principal business segments: Direct Attached
Storage ("DAS"), Storage Networking Solutions ("SNS") and Software. These hardware and
software products are found in high-performance networks, servers, workstations, and desktops
from the world's leading OEMs, and are sold through distribution channels to Internet service
providers, enterprises, medium and small businesses, and consumers. Since the time it went
public, Raptec has experienced rapid growth and consistently profitable operations. In early
2007, the company announced its plan to spin-off the software segment, subsequently
incorporated as Axio, Inc., in the form of a fully independent and separate company. Software
was Raptec's most profitable and fastest growing segment. By mid-2007 Raptec had completed
the initial public offering of approximately 15% of Axio's stock, and then distributed the
remaining Axio stock to Raptec's stockholders in a tax-free distribution. -(Refer to Case
Scenario 2)
1. Why would a successful firm like Raptec spin off its most promising business?
The best answers will begin by noting that both hardware and software are industries characterized by
fast cycle times, which requires management to be both focused and nimble. With this background,
students can then argue that the spin-off provides the management teams of Raptec and the newly
formed Axio with greater focus (on hardware and software respectively), better alignment of
employee incentives, and greater managerial accountability. The spin-off also provides Axio direct
access to capital markets.
2. Prior to the spin-off, how would you go about identifying the respective boundaries of
the Raptec and Axio businesses?
The purpose of this question is to point out how blurry the lines may be between businesses in a
diversified firm - the best answers will revolve around this point. While Raptec operated in three
business segments, this does not guarantee that each operated as independent organizations within
Raptec. In fact, Raptec likely benefited from tremendous operational and market synergies among its
three primary lines of business, and such synergies are typically accomplished through formal
coordination and integration mechanisms like organizational structure, systems, and processes. A
useful analogy here can be drawn to Palm, Inc., and its PDA product the Palm Pilot. Given that
consumers view the Palm PDA as a monolithic product (they don't think of it as separate hardware
and software, where would you begin to draw the dividing line if Palm wanted to split up its hardware
and software businesses?
3. What risks does Raptec run in spinning off Axio?
The best answers will point out that the spin-off strategy makes sense only to the extent that the
benefits described in the answer to question 1 considerably outweigh the costs arising from breaking
up the firm and its lost opportunities for within-firm synergies. If Raptec has been successful because
of its ability to uniquely couple hardware and software, along with the fact that it possesses inside
knowledge about the technological advances in each business, then breaking up the firm may actually
break-up and destroy a potential core competency. Also, once a firm has broken itself up into distinct
legal entities there is nothing to prevent one of the players from preying on the others' most profitable
related businesses. For instance, Axio may start moving into parts of the hardware business that, from
its inside experience with Raptec, it knows are highly profitable when combined with Axio's
proprietary software.

Case Scenario 3: Barracuda Inc. Barracuda Inc. has diversified beyond its early base as a lamp
fixture manufacturer into multiple hardware and plumbing fixture products that it sells to
professionals (i.e., plumbers and electricians) and through the large volume do-it-yourself (DIY)
stores like The Home Depot and Lowe's. While this successful growth has been achieved
primarily through acquisition, the company tends to let the acquired businesses run
independently. It has done so by looking to fragmented industries to acquire small firms with
efficient operations and good management teams. It then grows these businesses through a
combination of internal cash flow and debt, and directs new sales to the professional and DIY
channels. Barracuda has been particularly successful in the faucet segment, which it practically
reinvented though such technological innovations as the washerless faucet, and marketing
innovations like branding and good-better-best merchandising. Barracuda has leveraged this
merchandising strategy across its businesses and, coupled with the explosive growth of the DIY
channel, is spectacularly profitable with a net profit after tax (NPAT) of 18%. The firm's
management is looking to broaden its revenue base and has identified the home furnishings
business as sharing many characteristics with faucets, prior to Barracuda's entry into faucets. It
plans to enter this industry through large-scale acquisitions. The landscape of the U.S. home
furnishings manufacturing industry consists of many players, none with controlling share, and
serious issues of overcapacity. There are presently 2500 home furnishings firms, and only 600 of
those have over 15 employees. Average NPAT is between 4 and 5%, which also reflects the fact
that few firms have good managers. While the industry is still primarily comprised of single-
business family-run firms, which manufacture furniture domestically, imports are increasing at
a fairly rapid rate. Some of the European imports are leaders in contemporary design.
Relatively large established firms are also diversifying into the home furnishings industry via
acquisition. Supplier firms to the home furnishings industry are in relatively concentrated
industries (like lumber, steel, and textiles), and therefore typically offer fewer accommodations
to the small furniture manufacturers. Retailers, the intermediate customer of the home
furnishings industry, are becoming increasingly concentrated and the few large, successful
furniture companies actually have their own stores or have dedicated showrooms in the larger
department stores. Customers have many products to choose from, at many different price
points, and few home furnishing products beyond those of the larger companies have
established brands. Also, customers can switch easily among high and low-priced furniture and
other discretionary expenditures (spanning plasma TVs to the choice of postponing any
furniture purchase entirely). -(Refer to Case Scenario 3)
https://pdfslide.net/documents/sm5450d1d2b1af9f2c118b4cc2.html
1. Why would Barracuda consider acquisition as its preferred mode of entry into
furniture?
Barracuda considers acquisition as its preferred mode of entry into furniture to ensure the immediate
market presence in the furniture business and also to emphasize on the business diversification
strategy. As acquisition is the process to takes over the controlling interest of rivals,
so another concern was to gain market share and competencies, from the previous
successful growth was achieved primarily through acquisition gave them an experience of being
successful again by expanding them with appropriate strategy. Barracuda’s consideration of
acquisition is also being helpful to establish a powerful brand name in the furniture segment, but not
forgetting valuable knowledge, skill and experience of the employee which will transfer into
Barracuda from the acquired industry. Distribution channel of the acquiesced firm is an
added advantage to cut or reduce unnecessary cost for Barracuda. From the above argument it is well
understood that the Barracuda considering acquisition is a beneficial steps of the company to enter
into furniture.
2. Given the history of Barracuda, what guidelines would you suggest to management
regarding their acquisition strategy in the home furnishings industry?
Based on the case, Barracuda successful growth has been achieved primarily through acquisition, the
company tends to let the acquired businesses run independently shows a clearer picture of the good
and efficient management of Barracuda. However, there are several guidelines regarding their
acquisition strategy in the home furnishings strategy. First is when organization competes in a
no-growth or a slow-growth industry.In order to acquire small firm with efficient operation and good
management team, then start looking to fragmented industries. Second is when adding new, but
related, product would be significantly enhancing the sales of current product. As early
base of Barracuda Inc is lamp fixture manufacture into multiple hardware and plumbing fixture
product and has identified the home furnishing business as shaving characteristic
management team. Third is when new, but related, products could be offered at highly competitive
prices. Barracuda after do an acquisition, they can put the price of higher than their competitors. As
quality of the product are customer focused.
3. Given Barracuda's history, what threats does Barracuda face in entering the furniture
industry through acquisition?
Given Barracuda’s history of having high NPAT in other businesses, ne of the threats of entering the
home furnishing business is low NPAT as historically this industry haslower return due to lack of
good management and operation. Barracuda may not be able to run the home furnishing
segment indecently and this may require high attention from managers to integrate and build
the business and it may cause other pertinent issues to be neglected. Barracuda has been
successful with their technological and marketing innovation but the fact that lower
NPAT in the home furnishing segment may lead to shift of focus from innovation to financial
success rather than strategic control of other business. Innovation may be ignored as Barracuda tries
to achieve synergy and improve financial standing of the newly acquired.
4. What is the core business of Barracuda and identify the company strength.
The core business of Barracuda is lamp fixture manufacturer into multiple hardware and plumbing
fixture product that it sells to professional. The strength of Barracuda issuccessful in
the faucet segment, which is practically reinvented through such technological innovation
as the washerless faucet, and marketing innovation like branding and good better best
merchandising. Following with next strength which is strategy across its business and coupled with
explosive growth of DIY channel.
5. Describe at least 2 strategies used by Barracuda.
a) Product development: Product development is a strategy that seeks in increased sales
by improving or modifying present products or services. Product development usually entails
large research and development expenditure. As stated, Barracuda is seeking increased sales by
improving present on services or developing as new one.
b) Related diversification: Related diversification is about adding new but related product or services.
The firm management is looking to broaden its revenue base and has identified the
home furnishing business as sharing many characteristic with faucets, prior to Barracuda entry into
faucets. It can help the business will more efficient and good management team.
Quiz 8: International Strategy
http://www.staequiz.com/product/the-management-of-strategy-concepts-international-
edition-10th-edition-by-r-duane-ireland-test-bank/
https://quizlet.com/85345218/strategic-management-ch-8-essay-flash-cards/
https://www.studyblue.com/notes/note/n/chapter-8-international-strategy/deck/7118953
1. What are the motives for firms to pursue international diversification? What are the
four basic benefits firms can derive by moving into international markets?
One reason is to extend the life cycle of the firm's products. Gaining access to needed and potentially
scarce resources is another reason. There is also pressure for global integration of operations, driven
by growing universal product demand. Companies also want to take advantage of opportunities to
better use rapidly developing technologies such as the Internet and mobile applications, which permit
greater integration of trade, capital, and culture. Finally, the potential of large demand for goods and
services for people in emerging markets is an important incentive. When firms successfully move into
international markets, they can experience: increased market size, economies of scale and learning,
and location advantages.
2. What are the four factors that provide a basis for international business-level strategies?
Firms derive three basic benefits by successfully using international strategies: (1) increased market
size, (2) economies of scale and learning, and (3) advantages of location. Increased market size is
achieved by expansion beyond the firm's home country. International expansion increases the number
of potential customers a firm may serve. Starbucks is a firm that has increased its market size through
international expansion (Opening Case). Other firms such as Coca Cola and PepsiCo have moved into
international markets primarily because of limited growth opportunities in their domestic markets.
Economies of scale and learning is a second benefit. Leveraging a technology beyond the home
country allows for more units to be sold and initial investments recovered more quickly. Rivals
Airbus and Boeing have multiple manufacturing facilities and outsource some activities in order to
gain scale advantages. Lastly, advantages of location can be realized through internationalization.
These advantages include access to low-cost labor, critical resources, or customers.
3. Discuss the three international corporate-level strategies. On what factors are these
strategies based?
International corporate strategy focuses on the scope of a firm's operations through both product and
geographic diversification. The three basic international corporate- level strategies vary on the need
for local responsiveness to the market and the need for global integration. The multidomestic strategy
focuses on competition within each country in which the firm operates. Firms employing a
multidomestic strategy decentralize strategic and operating decisions to the strategic business units
operating in each country so business units can customize their goods and services to the local market.
The use of global integration in this strategy is low. The global strategy assumes more standardization
of product demand across country boundaries. Therefore, competitive strategy is centralized and
controlled by the home office, placing high emphasis on global integration of operations. The
strategic business units in each country are interdependent and the home office integrates these
businesses. The firm offers standardized products across country markets. It emphasizes economies of
scale and the opportunity to use innovations developed in one nation to other markets. The
transnational strategy seeks to achieve both global efficiency (through global integration) and local
responsiveness. This strategy is difficult to implement. One goal requires global coordination while
the other requires local flexibility. Flexible coordination builds a shared vision and individual
commitment through an integrated network. The effective implementation of the transnational
strategy often produces higher performance than either of the other corporate-level strategies.
4. Identify and describe the modes of entering international markets. What are their
advantages and disadvantages?
Firms may enter international markets in any of five ways: exporting, licensing, forming strategic
alliances, making acquisitions, and establishing new, wholly owned subsidiaries (greenfield ventures).
● Most firms, particularly small ones, begin with exporting (marketing and distributing their
products abroad). This involves high transportation costs and possibly tariffs. An exporter has less
control over the marketing and distribution of the product than in other methods of entering the
international market. In addition, the exporter must pay the distributor or allow the distributor to add
to the product price in order to offset its costs and earn a profit. In addition, the strength of the dollar
against foreign currencies is a constant uncertainty. But, the advantages are that the company does not
have the expense of establishing operations in the host countries. The Internet makes exporting easier
than in previous times.
● Licensing (selling the manufacturing and distribution rights to a foreign firm) is also popular
with smaller firms. The licenser is paid a royalty on each unit sold by the licensee. The licensee takes
the risks and makes the financial investments in manufacturing and distributing the product. It is the
least costly way of entering international markets. It allows a firm to expand returns based on a
previous innovation. But there are disadvantages. Licensing provides the lowest returns, because they
must be shared between the licensee and the licensor. Licensing gives the licenser less control over
the manufacturing and marketing process. There is the risk that the licensee will learn the technology
and become a competitor when the original license expires. If the licenser later wishes to use a
different ownership arrangement, the licensing arrangement make create some inflexibility.
● Strategic alliances involve sharing risks and resources with another firm in the host country.
The host country partner knows the local conditions; the expanding firm has the technology or other
capabilities. Both partners typically enter an alliance in order to learn new capabilities. The
partnership allows the entering firm to gain access to a new market and avoid paying tariffs. The host-
country firm gains access to new technology and innovative products. Equity-based alliances are more
likely to produce positive gains. Alliances work best in the face of high uncertainty and where
cooperation is needed between partners and strategic flexibility is important. But, many alliances fail
due to incompatibility and conflict between the partners. Cross- border acquisitions provide quick
access to a new market, but they are expensive and involve complex negotiations. Cross-border
acquisitions have all the problems of domestic acquisitions with the complication of a different
culture, legal system and regulatory requirements. Acquisitions are expensive and usually involve
debt- financing.
● The most expensive and risky means of entering a new international market is through the
establishment of a new, wholly owned subsidiary or greenfield venture. Alternatively, it provides the
advantages of maximum control for the firm and, if successful, potentially the greatest returns as well.
This alternative is suitable for firms with strong intangible capabilities and/or proprietary technology.
The risks are high because of the challenges of operating in an unfamiliar environment. The company
must build new manufacturing facilities, establish distribution networks, and learn and implement
new marketing strategies.
5. Discuss the effect of international diversification on a firm's returns.
Research shows that returns vary as the level of diversification increases. At first, returns decrease,
then as the firm learns to manage the diversification, returns increase. But, as diversification increases
past some point, returns level off and become negative. Firms that are broadly diversified in
international markets usually receive the most positive stock returns, especially when they diversify
geographically into core business areas. International diversification can lead to economies of scale
and experience, location advantages, increased market size, and the potential to stabilize returns
(which reduces the firm's overall risk). International diversification improves a firm's ability to
increase returns from innovation before competitors can overcome the initial competitive advantage.
In addition, as firms move into international markets, they are exposed to new products and processes
that stimulate further innovation. The amount of international diversification that can be managed
varies from firm to firm and according to the abilities of the firm's managers. The problems of central
coordination and integration are mitigated if the firm diversifies into more friendly countries that are
geographically and culturally close.
6. Identify and describe the major risks of international diversification.
International diversification carries multiple risks. The major risks are political and economic.
Political risks are related to governmental instability and to war. Instability in a government creates
economic risks and uncertainty created by government regulation. Governmental instability can result
in the existence of many potentially conflicting legal authorities, corruption, and the risk of
nationalization of company assets. Economic risks are related to political risks. Economic risks
include the increased trend of counterfeit products and the lack of global policing of these products.
Another economic risk is the perceived security risk of a foreign firm acquiring firms that have key
natural resources or firms that may be considered strategic in regard to intellectual property. In
addition, differences in and fluctuations of the value of different currencies is another economic risk.
The security risk created by terrorism prevents U.S. firms from investing in some regions. The
relative strength or weakness of the dollar affects international firms' competitiveness in certain
markets and their returns.
Case Scenario 1: Blast Furnace, Inc., (BFI) Blast Furnace, Inc., (BFI) provides customized
development of automated rich-media applications, and scalable solutions that allow media and
entertainment companies, as well as enterprises and government organizations, to deploy,
manage and distribute video content on IP-based networks. The company was founded in 1997
and went public in 2004; its stock trades on the NASDAQ under the ticker BLST. While
providing solutions to a variety of firms and industries in North America, BFI has experienced
its fastest growth with the security products that it designs and sells to the U.S. government and
U.S. government agencies. This growth is based on its propriety VUE software, which is a
complete identification solution for capturing, analyzing, and managing multi-biometric
information. Proprietary analysis algorithms aggregate and cross-compare multiple biometrics
to increase accuracy and lessen dependence on single identification techniques. Additionally,
specialized encoding techniques reduce file size and increase analysis and response times. VUE
supports a wide range of applications ranging from ID issuance and verification to gated entry
screening for border patrol, airports, government buildings, and corporations. Essentially, VUE
is able to sift through massive amounts of digitized multimedia files to create a unified ID
dossier of an individual and then identify those individuals rapidly anywhere in the data. Such a
capability is of great interest to security organizations, particularly since the World Trade
Center bombing, because it allows the user to identify suspects within minutes on a real-time
basis anywhere digitized media is being created (like that created by the hidden and visible
surveillance cameras in airline terminals, banks, ATMs, and other public locations). Once a
suspect is entered into the system, the software is capable of scanning all data sources
automatically and without stop. For two years, BFI has had this market to itself but now two
new entrants, a Belgian start-up and the subsidiary of a Finnish telecom firm, are staking out
positions in large non-U.S. markets like Europe and Asia. BFI's management fears that if it
limits its efforts to North America, then these aggressive competitors may eventually develop
strongholds in other markets from which they can launch successful attacks on BFI's home turf.
-(Refer to Case Scenario 1)
1. Should BFI expand its operations outside of North America?
2. Assume that BFI has chosen international expansion. How quickly should it move? Which
activities would you recommend BFI to internationalize first?
3. How would you evaluate which country or countries BFI should enter first?

Case Scenario 2: Heartsong LLC. Heartsong LLC is a designer and manufacturer of


replacement heart valves based in Peoria, Illinois. While a relatively small company in the
medical devices field, it has established a worldwide reputation as the provider of choice of
high-quality, leading-edge artificial heart valves. Most of its products are sold to large regional
hospital systems and research hospitals around the world, though primarily to customers in the
U.S. and Europe. Specialty heart centers are another emerging, but fast-growing market for its
valves. Heartsong has recently embarked on an expansion strategy that requires it to increase
its volume, which in turn will demand more component parts than it can source domestically -
both from an economic and volume standpoint. The firm has determined that such growth is
only viable if it produces these parts itself overseas for a lower cost, or outsources the
production entirely to a joint venture it establishes with a local manufacturer, which could both
produce the parts more cheaply and in higher volumes. It is considering starting up an owned
production facility in Luxembourg, or seeking a joint venture with a precision manufacturer in
China. -(Refer to Case Scenario 2)
http://www.ziwojieshao.org/fanwen/359912.html
https://issuu.com/mcdonaldmcdonald/docs/strategic-management-concepts-and-c

1. What opportunities and threats might Heartsong be exposing itself to via the
Luxembourg expansion proposal?
The best answers will begin by noting that Heartsong has the capacity to design leading-edge medical
products and then take these designs and turn them into reliably manufactured, high-quality
replacement heart valves. Thus, basic R&D and quality precision manufacturing are likely to be
critical value-creating facets for this firm.
2. What opportunities and threats might Heartsong be exposing itself to via the China
expansion proposal?
The best answers will start by observing that the scenario suggests that Heartsong needs to grow if it
is going to continue being competitive and successful. However, Heartsong is also capital constrained
and an outsourcing arrangement with EdFex allows it to more efficiently manage this significant
aspect of its cost base (inventory and delivery). This outsourcing solution would be ideal if it would
allow Heartsong to maintain a centralized warehouse with heart valve inventory in major population
centers, instead of its present practice of carrying inventory on the shelves of each of its hospital
customers. As a result, Heartsong could grow its market presence, while more efficiently managing
the need to have heart valves available on short notice.
3. Which option would you recommend?
The best answers will develop the theme that the EdFex outsourcing arrangement is truly likely to be
win-win. With the arrangement in place, Heartsong is able to devote its financial, human capital, and
managerial resources to basic R&D and quality precision manufacturing; and, EdFex does what it
does best in logistics. Moreover, it is hard to contemplate that EdFex would ever think of entering the
heart valve industry – thus, EdFex does not pose a direct threat as a future competitor. It does
however pose an indirect threat to Heartsong to the extent it can hold the firm hostage, and extract
exorbitant fees for its logistic services.
Quiz 9: Cooperative Strategy
https://quizlet.com/145599064/final-part-2-multiple-choice-only-9-flash-cards/
1. Identify and define the different types of strategic alliances.
Strategic alliances are cooperative strategies between firms whereby resources and capabilities are
combined to create a competitive advantage. All strategic alliances require firms to exchange and
share resources and capabilities to co-develop or distribute goods or services. The three basic types of
strategic alliances are: (1) joint ventures, where a legally independent company is created by at least
two other firms, with each firm usually owning an equal percentage of the new company; 2) equity
strategic alliances, whereby partners own different percentages of equity in the new company they
have formed; and (3) non equity strategic alliances, which are contractual relationships between firms
to share some of their resources and capabilities. The firms do not establish a separate organization,
nor do they take an equity position. Because of this, non equity strategic alliances are less formal and
demand fewer partner commitments than joint ventures and equity strategic alliances. Typical forms
are licensing agreements, distribution agreements and supply contracts.
2. Explain the rationales for a cooperative strategy under each of the three types of basic
market situations (i.e., slow, standard, and fast cycles).
In slow-cycle markets (markets that are near-monopolies), firms cooperate with others to gain entry
into restricted markets or to establish franchises in new markets. Slow-cycle markets are rare and
diminishing. Cooperative strategies can help firms in (presently) slow-cycle markets make the
transition from this relatively sheltered existence to a more competitive environment. In standard-
cycle markets (which are often large and oriented toward economies of scale), firms try to gain access
to partners with complementary resources and capabilities. Through the alliance, the firms try to
increase economies of scale and market power. In fast-cycle markets (characterized by instability,
unpredictability, and complexity) sustained competitive advantages are rare, so firms must constantly
seek new sources of competitive advantage. In fast-cycle markets alliances between firms with excess
resources and capabilities and firms with promising capabilities who lack resources help both firms to
rapidly enter new markets.
3. Identify the four types of business-level cooperative strategies and the advantages and
disadvantages of each.
Through vertical and horizontal complementary alliances, companies combine their resources and
capabilities in ways that create value. Vertical complementary strategic alliances result when firms
creating value in different parts of the value chain combine their assets to create a competitive
advantage. Vertical complementary strategies have the greatest probability of being successful
compared with other types of cooperative strategies. But firms using this type of alliance need to be
wise in how much technology they share with their partners. Vertical complementary alliances rely
heavily on trust between partners to succeed. Horizontal complementary strategic alliances are
developed when firms in the same stage of the value chain combine their assets to create additional
value. Usually they are formed to improve long-term product development and distribution
opportunities. Horizontal complementary strategies can be unstable because they often join highly
rivalrous competitors. In addition, even though partners may make similar investments, they rarely
benefit equally from the alliance. The competition response strategy involves alliances formed to react
to competitors' actions. Usually they respond to strategic, rather than tactical, actions because the
alliances are difficult to reverse and expensive to operate. The uncertainty-reducing strategy is used to
hedge against risk and uncertainty, such as when entering new product markets or in emerging
economies. Both of these strategies are less effective in the long-run than the complementary alliances
which are focused on creating value. Competition reducing (collusive) strategies are often illegal.
There are two types of collusive competition reducing strategies: explicit collusion and tacit collusion.
Explicit collusion exists when firms directly negotiate production output and pricing agreements to
reduce competition. These are illegal in the U.S. and in most developed economies. Tacit collusion
exists when several firms in an industry indirectly coordinate their production and pricing decisions
by observing each other's competitive actions and responses. Both types of collusion result in lower
production levels and higher prices for consumers.
4. Identify the three types of corporate-level cooperative strategies.
A diversifying strategic alliance allows firms to share some of their resources and capabilities to
diversify into new product or market areas. A synergistic strategic alliance allows firms to share some
of their resources and capabilities to create economies of scope. These alliances create synergy across
multiple functions or multiple businesses between partner firms. Franchising is a strategy in which the
franchisor uses a contractual relationship to describe and control the sharing of its resources and
capabilities with franchisees. A franchise is a contract between two independent organizations
whereby the franchisor grants the right to the franchisee to sell the franchisor's product or do business
under its trademarks in a given location for a specified period of time.
5. Why are cooperative strategies often used when firms pursue international strategies?
What are the advantages and disadvantages of international cooperative strategies?
A cross-border strategic alliance is an international cooperative strategy in which firms headquartered
in different nations combine some of their resources and capabilities to create a competitive
advantage. The typical reasons follow: 1) In general multinational firms outperform firms operating
only on a domestic basis. Firms may be able to leverage core competencies developed domestically in
other countries. 2) Limited domestic growth opportunities pushes firms into international expansion.
3) Some governments require local ownership in order for foreign firms to invest in businesses in
their countries, which requires foreign firms to ally with local firms. 4) Local partners often have
significantly more information about factors contributing to competitive success such as local
markets, sources of capital, legal procedures, and politics, which makes an alliance useful for a
foreign firm. 5) Cross-border alliances can help firms transform themselves or better use their
competitive advantages surfacing in the global economy. On the negative side, cross-border alliances
are more complex and risky than domestic strategic alliances.
6. Identify and define the two different types of network strategies.
A network cooperative strategy is a cooperative strategy wherein several firms form multiple
partnerships to achieve shared objectives. Stable alliance networks (primarily found in mature
industries) usually involve exploitation of economies of scale or scope. In this type of network, the
firms try to extend their competitive advantages to other settings while continuing to profit from
operations in their core industries. Dynamic alliance networks (witnessed mainly in rapidly changing
industries) are used to help a firm keep up when technologies shift rapidly by stimulating product
innovation and successful market entries. Dynamic alliance networks explore new ideas and typically
generate frequent product innovations with short product life cycles.
7. Identify the competitive risks associated with cooperative strategies.
Cooperative strategies are not risk free strategy choices; as many as 70% fail. If a contract is not
developed appropriately and fails to avert opportunistic behavior, or if a potential partner firm
misrepresents its competencies or fails to make available promised complementary resources, failure
is likely. Furthermore, a firm may make investments that are specific to the alliance while the partner
does not. This puts the investing firm at a disadvantage in terms of return on investment. The core of
many failures is the lack of trustworthiness of the partner(s) who act opportunistically.
8. Describe the strategic management approaches to managing alliances.
The ability to effectively manage competitive strategies can be one of a firm's core competencies.
There are two basic approaches to managing competitive alliances. Cost minimization leads firms to
develop protective formal contracts and effective monitoring systems to manage alliances. Its focus is
to prevent opportunistic behavior by the partner(s). Opportunity maximization is intended to
maximize value creation opportunities. It is less formal and places fewer constraints on partner
behaviors. But, identifying trustworthy partners is the key to this second approach. If (well-founded)
trust is present, monitoring costs are lowered and opportunities will be maximized. Trust is more
difficult to establish between international partners. Ironically, the cost minimization approach is
more expensive to implement and to use than the opportunity maximization approach.
Case Scenario 1: Norning International Norning International (NI) states that both its past
successes and future growth strategies are based on an evolving network of wholly owned
businesses and joint ventures around its core competency in glass making. Through their
alliances and owned divisions they compete in four global business sectors: Specialty Glass and
Materials (including materials for HDTV and LCD displays), Consumer Housewares (including
microwavable dishware), Laboratory Sciences Products and Services (test tubes, testing
equipment, and drug trials testing), and Communications (fiber optics and related
technologies). Per the company's annual report, "binding all four sectors together is the glue of
a commitment to leading edge glass making technologies, shared resources, and dedication to
total quality." Each sector is composed of divisions, subsidiaries and alliances. However, the
central role played by alliances is demonstrated by the fact that the combined revenue of its 30-
some alliances is more than double that of NI on its own. Most of the alliances provide NI with
access to particular geographic markets, industries, or channels, although an increasing number
of alliances involve both market access and technological development. -(Refer to Case Scenario
1)
1. Why would a company like NI place such emphasis on alliances as a growth vehicle?
There are many components that are involved in glassmaking technologies, so in order to gain more
resources and produce a better product to consumers, it is essential to form alliances to merge their
different segments for efficiently. Also, for firms like NI who don't have enough capital to acquire a
firm, they can form a less capital intensive agreement to partner with another firm.
Moreover, if NI is able to retain its focus and leading edge expertise in advanced glass-making
techno-logies, while at the same time realize repeated successes and technological breakthroughs via
alliances, then its ability to partner and manage multiple alliances may truly become a strategic asset.
2. What risks arise from a strategy based on such a "network of alliances"?
The best answers will start by noting that NI’s historic success (via its core competency in glass
making) is likely to have been based in a strong commitment to basic research and development.
Multiple partnerships, in and of themselves, can lead to the dilution of management's attention and
investment in this core, or lead to a less well defined core. Given that such basic R&D is the “glue”
holding NI together, such dilution may make that glue at the hub of its strategy wheel come unstuck.
Exacerbating this threat is the possibility that NI’s fascination with alliances may become a crutch
where alliances displace its internal strengths and become an end in and of themselves. Alliance
partners are also a threat to the extent they are able to learn more from NI about its technologies, and
eventually undermine or leapfrog NI technological competitive advantages.
3. NI appears to be managing a large number of alliances. What criteria should it use to
exit particular alliances?
The best answers will develop a set of company rules of thumb for exiting alliances, and even some of
NI’s owned businesses. For instance, given the fundamental role played by the need to emphasize ba-
sic R&D in advanced glass-making technologies, students may first start by observing that NI should
likely exit those businesses that are highly service-based (like some of its laboratory services
businesses), or do not require leading edge glass-making expertise (like consumer glass-based
products), since neither of these businesses require or contribute to NI’s understanding of new glass-
making technologies. Consequently, NI should exit the alliances related to those particular segments
because they don’t help to reinvigorate its technological core. Beyond this rule of thumb, students
may then push NI to exit those alliances where there is the least opportunity to learn from its partners,
and again reinvigorate and further enhance its core in advanced glass making.
Case Scenario 2: ERP Inc ERP, Inc., (ERPI) is a leading provider of enterprise integration
software (EIS). EIS essentially allows a firm to connect and integrate processes across all
aspects of its business. To fuel its dramatic growth, ERPI has focused its organization entirely
on product development (software programming for a suite of EIS products) and selling
(making the sale and then moving onto a new target), while outsourcing the installation and
consulting aspects to the world's largest accounting firms. This also makes ERPI basically a
"product company," whereas most competitors like Oracle and PeopleSoft in its market space
operate as "solutions companies." One benefit of this focused strategy is that ERPI's product is
generally recognized as being 200% to 300% better than competitors' software, and thus
adopters are thus likely to have a one to two year advantage. In further contrast to the
competition, ERPI has used its partnerships with the accounting firms to deliver a turn-key
solution, and has focused this solution on a market comprised of the world's largest, global
manufacturers and consumer product companies. The accounting firms, in turn, coordinate a
comprehensive collection of hardware, operating systems, and complementary software firms.
Installation and related consulting for EIS typically cost between $100 and $200 million, with
the ERPI software component accounting for only about 20% of the installed cost (the
remaining 80% is spent on the actual installation, not counting the value of the customer's
time). To incentivize the accounting firms to help sell its product (since, at least initially, the
accounting firms had better reputations and controlled access to the target customers), ERPI
told its partners that it will never enter the installations and consulting side of the business
(aside from installation and consulting that ERPI does as part of its software support). Dangling
such a large carrot in front of the accounting firms provided the continuing benefit of
encouraging their continued support of ERPI with their customers. -(Refer to Case Scenario 2)
1. Given that software systems like EIS are very complex, and quality is largely a function
of the related installation and consulting processes, how can ERPI control quality and
ultimately protect the reputation of its product (and its name) when it has ultimately outsourced
installation to its partners?
The best answers will simply walk through the respective columns in Table 3.5 and reach the
conclusion that, at least in the near term, ERPI has a sustainable competitive advantage. Its EIS
software is valuable given that it is 200% to 300% better than competitors’ products. It is similarly
rare and nonsubstitutable since it is proprietary, and currently has a two-year lead on the alternatives.
A similar rationale can be invoked to support the argument that ERPI’s capabilities in software
programming are going to be costly to imitate. A competitor would have to hire a similar workforce
or acquire a company that currently occupies the same market space. This strong position is further
bolstered by the fact that a large percentage of the market is voting with its feet in favor of ERPI.
2. After managing this network of alliances for several years, what new strategic assets has
ERPI developed?
This question asks students to take a more dynamic perspective of potentially valuable resources that
companies and their customers create together, but that the company itself can exploit (a perfect
example of a co-specialized asset) The best answers will begin by observing that if ERPI has focused
histonically on transactions (making the sale), then it has given hittle explicit consideration to
customers as long-term relationships beyond the need to provide technical support (lifetime value of a
customer beyond the first sale). Shifting attention to ERPI installations as relationships suggests that
the company now has a customer list to die for. This list is especially valuable since (1) the target
companies have invested upwards of $200 million in ERPI proprietary systems and, (2) once
installed, given the pervasive nature of EIS systems, those target firms are unlikely to simply switch
to another system.
3. Imagine that ERPI has saturated the large-firm market for its products, competitors are
undermining its technological advantage, and ERPI needs to look to new markets for revenue.
Its CEO has suggested that it start selling its software down-market to middle-market
companies, and at the same time enter the consulting and installation side of the business for
this target market. What are the risks and opportunities of such a strategy?
The best answers will build on the basic notions developed in response to question 4., Students will
argue that ERPI's competitive advantage is sustainable as long as its technology continues to define
the leading edge of EIS products and that substitute solutions do not encroach much on its two-vear
lead. However, and as is consistent with most high-technology markets, as students pick apart ERPI's
capabilities following the categories in Table 3.5 they should begin to see that sustained competitive
advantage in this particular market space may be difficult, particularly given the presence of large
aggressive competitors like Oracle and Microsoft, which are intent on gaining a presence in the EIS
market
Case Scenario 3: Bunnywac. Bunnywac is a global producer and seller of batteries for consumer
electronics products (radios, flashlights, toys, etc.), and competes primarily with its larger rivals
by providing battery products equal in performance at a lower price. The worldwide battery
industry suffers from issues of overcapacity and commoditization, brand segmentation and
proliferation, the growing strength of global retailers, and the low-cost threat of new entrants
from Asia. Thus, the ability to provide dependable batteries at a very low cost is essential to
survival in this industry. Bunnywac has grown quickly into one of the leading players in the
battery industry primary through horizontal acquisitions financed by a recent successful IPO,
and is now counted among the top four companies in North and Latin America. Its presence in
Europe and Latin America is negligible. While its market presence and brand is generally
strong and market share is growing, Bunnywac has entered into an alliance to obtain the core
technologies of its batteries. Bunnywac does not actually own the technology that makes its
batteries work. This approach has provided Bunnywac a cost advantage since it has not had to
invest in basic R&D and has very little R&D infrastructure. This technology is licensed from
Mats (which has 200 engineers dedicated to moving the technology forward), one of Japan's
largest technology-based holding companies (like Sharp or Canon). Mats also sells batteries
under the Pandemonium brand and commands over 50% of the market share of Asian
countries. Mats' market share in other global markets is negligible and its efforts at growing its
branded battery share in the North America, Latin America, and Europe has been severely
frustrated in recent years. While Mats is very large compared to Bunnywac, the battery
technology and battery business are relatively tiny relative to Mats' other technology-based
businesses. Bunnywac's decade-long licensing agreement with Mats for the essential battery
technology expires in one year; there are no obvious substitute providers of this technology. -
(Refer to Case Scenario 3)
1. What should be Bunnywac's primary concerns about its lapsing technology contract
with Mats?
The best discussions will draw out near and longer-term concerns for Bunnywac’s prospects, as well
as speculation as to Mats’ long-term strategic intent. Near-term, Bunnywac needs to have access to a
reasonably priced technology if it is to continue competing on a low-cost basis. Also, since it is a
public company, the capital markets are likely to begin showing concern for Bunnywac’s prospects if
no replacement technologies are identified, which in turn will hamper the firm’s ability to grow
further through acquisition. Medium-term, if a new contract is negotiated, Bunnywac needs to be
concerned with the price it will now pay, since Mats can be construed to be in the more advantageous
bargaining position. If Bunnywac is determined to remain independent, then it should only pay such
higher costs if it has a parallel near-term strategy of developing its own technological competencies
(which are admittedly very hard and costly to develop) or another source for the technology at a lower
price and on a long-term basis. Finally (and perhaps first!), Bunnywac needs to assess Mats’ strategic
intent with regard to Bunnywac. Does Mats view, and is hence positioning for, Bunnywac as (a) a
valued-long term partner, (b) a potential future competitor, or (c) a near-term cheap acquisition target
as a vehicle for gaining a quick foothold in the branded battery markets?
2. What should Bunnywac's strategy be with regard to the lapsing technology contract?
Following the discussion of its concerns, the best answers will have several possible steps. The first
step should be for Bunnywac to determine Mats’ long-term strategic intent. Is Mats primarily
interested in retaining the technology? Would they sell the technology outright? Is their primary intent
to forward integrate into all global markets through their own or acquired brands? These answers
determine whether or not Mats’ intent is to see Bunnywac as a viable, independent avenue for its
technologies, or simply as a potential future competitor, or something in between. For instance, if
Mats sees Bunnywac as adding value in terms of brand and channel management capabilities, but is
not interested in extending the technology alliance, then perhaps Bunnywac may actually consider
negotiating its own sale to Mats while its value is still relatively high.
Quiz 10: Corporate Governance
https://www.studocu.com/en-us/document/rutgers-university/financial-analysis-and-
corporate-governance/past-exams/corporate-governance-chapter-10-exam-
answers/2268990/view
1. What is corporate governance and how is it used to monitor and control managers'
decisions?
Corporate governance is the relationship among stakeholders that is used to determine and control the
firm's strategic direction and its performance. Effective governance that aligns top-level managers'
interests with shareholders' interests can produce a competitive advantage for the firm. Corporate
governance includes oversight in areas where there are conflicts of interest among major stakeholders,
including the election of directors, supervision of CEO pay, and the organization's overall structure
and strategic direction. Three internal governance mechanisms (ownership concentration, the board of
directors, and executive compensation) and an external mechanism (the market for corporate control)
are used in U.S. corporations. Unfortunately, corporate governance mechanisms are not always
successful.
2. Discuss the effect of the separation of ownership and control in the modern corporation.
Ownership is typically separated from control in the large U.S. corporation. Owners (principals) hire
managers (agents) to make decisions that maximize the value of their firm. As risk specialists, owners
diversify their risk by investing in an array of corporations. As decision-making specialists, top
executives are expected by owners to make decisions that will result in earning above-average returns
for which they are compensated. Thus, the typical corporation is characterized by an agency
relationship that is created when one party (the firm's owners) hires and pays another party (top
executives) to use decision-making skills. Since owners may not possess the specialized skill to run a
large company, delegating these tasks to managers should produce higher returns for owners.
3. Define the agency relationship and managerial opportunism and discuss their strategic
implications.
The separation of owners and managers creates an agency relationship. An agency relationship exists
when a principal hires an agent as a decision-making specialist to perform a service. Some problems
that result from the agency relationship between owners and managers include the potential for a
divergence of interests and a lack of direct control of the firm by shareholders. Managerial
opportunism is the seeking of self-interest with guile. It is both an attitude and a set of behaviors,
which cannot be perfectly predicted from the agent's reputation. Top executives may make strategic
decisions that maximize their personal welfare and minimize their personal risk, such as excessive
product diversification. Decisions such as these prevent the maximization of shareholder wealth,
which is supposed to be the top executives' priority. Although shareholders implement corporate
governance mechanisms to protect themselves from managerial opportunism, these mechanisms are
imperfect. Agency costs include the costs of managerial incentives, monitoring costs, enforcement
costs, and the individual financial losses incurred by principals (owners of the firm) because
governance mechanisms cannot guarantee total compliance by the agents (managers).
4. Define the three internal corporate governance mechanisms and how they may be used
to control and monitor managerial decisions.
The three internal corporate governance mechanisms are: ownership concentration, the board of
directors, and executive compensation. Ownership concentration is based on the number of large-
block shareholders and the percentage of shares they own. With significant ownership percentage,
institutional investors, such as mutual funds and pension funds, are often able to influence top
executives' strategic decisions and actions. Thus, unlike diffuse ownership, which tends to result in
relatively weak monitoring and control of managerial decisions, concentrated ownership produces
more active and effective monitoring of top executives. An increasingly powerful force in corporate
America, institutional owners are actively using their positions of concentrated ownership in
individual companies to force managers and boards of directors to make decisions that maximize a
firm's value. These owners (e.g., CalPERS) have caused poorly performing CEOs to be ousted from
the firm. The board of directors, elected by shareholders, is composed of insiders, related outsiders,
and outsiders. The board of directors is a governance mechanism shareholders expect to run the firm
in such a ways as to maximize shareholder wealth. Outside directors are expected to be more
independent of a firm's top executives than are those who hold top management positions within the
firm. A board with a significant percentage of insiders tends to be weak in monitoring and controlling
management decisions. Boards of directors have been criticized for being ineffective, and there is a
movement to more formally evaluate the performance of boards and their individual members.
Executive compensation is a highly visible and often criticized governance mechanism. Salary,
bonuses, and long-term incentives such as stock options are intended to reward top executives for
aligning their goals with the interests of shareholders. A firm's board of directors has the
responsibility of determining the degree to which executive compensation succeeds in controlling
managerial behavior. But, it is difficult to evaluate top executives' performance, andso executive
compensation tends to be linked to financial measures which do not necessarily reflect the
effectiveness of the executive's decision on long-term shareholder outcomes. In addition, many
external factors affect the performance of a firm. Moreover, performance incentive plans can be
subject to management manipulation. Consequently, executive compensation is a far-from-perfect
governance mechanism.
5. Discuss the difficulties in establishing performance-based compensation plans for
executives.
Executive compensation, especially long-term incentive compensation, is complicated.First, the
strategic decisions made by top-level managers are typically complex and nonroutine; as such, direct
supervision of executives is inappropriate for judging the quality of their decisions. Because of this,
there is a tendency to link the compensation of top-level managers to measurable outcomes such as
financial performance.Second, an executive's decision often affects a firm's financial outcomes over
an extended period of time, making it difficult to assess the effect of current decisions on the
corporation's performance. In fact, strategic decisions are more likely to have long-term, rather than
short-term, effects on a company's strategic outcomes. Third, a number of other factors affect firm
performance. Unpredictable economic, social, or legal changes make it difficult to discern the effects
of strategic decisions. Thus, although performance-based compensation may provide incentives to
managers to make decisions that best serve shareholders' interests, such compensation plans alone are
imperfect in their ability to monitor and control managers.Although incentive compensation plans
may increase firm value in line with shareholder expectations, they are subject to managerial
manipulation. For instance, annual bonuses may provide incentives to pursue short-run objectives at
the expense of the firm's long-term interests. Supporting this conclusion, some research has found that
bonuses based on annual performance were negatively related to investments in R&D, which may
affect the firm's long-term strategic competitiveness. Although long-term performance-based
incentives may reduce the temptation to underinvest in the short run, they increase executive exposure
to risks associated with uncontrollable events, such as market fluctuations and industry decline. Long-
term incentives may not be highly valued by a manager: thus, firms may have to overcompensate
managers when they use long-term incentives.
6. Describe the market for corporate control and its implications for organizations.
The market for corporate control is composed of individuals and firms who buy ownership positions
in (e.g., take over) potentially undervalued firms to form a new division in an established firm or to
merge the two previously separate firms. The target firm's top management team is usually replaced
because it is assumed to be partly responsible for formulating and implementing the strategy that led
to poor firm performance. The market for corporate control is (supposedly) triggered by low corporate
performance by a firm relative to competitors in its industry. Thus, the market for corporate control
should act as a control mechanism for corporate governance that leads to the replacement of under-
performing executives. But, the market for corporate control is not an efficient governance
mechanism because in reality many of the firms taken over have above-average performance. Hostile
takeovers, on the other hand, are typically triggered by poor performance. Some managers have
sought to buffer themselves from the effect of the market for corporate control (hostile takeovers) by
instituting golden parachutes that will pay the managers significant extra compensation if the firm is
taken over. Those and other takeover defenses are intended to increase the costs of mounting a
takeover and reducing the managers' risk of losing their jobs. Examples of takeover defenses include
asset restructuring, changes in the financial structure of the firm, reincorporation in another state, and
greenmail. These defense tactics are controversial and the research on their effectiveness is
inconclusive. Most institutional investors oppose them.
7. Briefly compare and contrast corporate governance in the U.S., Germany, and Japan,
and China.
Corporate governance structures used in Germany and Japan differ from each other and from the ones
used in the United States. Historically, the U.S. governance structure has focused on maximizing
shareholder value. Banks have been at the center of the German corporate governance structure,
because as lenders, banks become major shareholders in the firms. Shareholders usually allow the
banks to vote their ownership positions, so banks have majority positions in many German firms. The
German system has other unique features. For example, German firms with more than 2,000
employees are required to have a two-tier board structure, separating the board's management
supervision function from other duties that it would normally perform in the United States (e.g.,
nominating new board members). Historically, German executives have not been dedicated to the
maximization of shareholder value, because private shareholders rarely have major ownership in
German firms, nor do larger institutional investors play a significant role.
Attitudes toward corporate governance in Japan are affected by the concepts of obligation, family, and
consensus. Japan continues to follow a bank-based financial and corporate governance structure
compared to the market-based financial and corporate governance structure in the United States. In
addition, Japanese firms belong to keiretsu, groups of firms tied together by cross-shareholding. In
many cases, the main-bank relationship of the firm is part of a keiretsu. However, the influence of
banks in monitoring and controlling managerial behavior and firm outcomes is beginning to lessen
and a minor market for corporate control is emerging.Chinese corporate governance has become
stronger in recent years. There has been a decline in equity held in state-owned enterprises, but the
state still dominates the strategies employed by most firms. Firms with higher state ownership tend to
have lower market value and more volatility in those values over time. In a broad sense, the Chinese
governance system has been moving toward the Western model in recent years. For example, YCT
International recently announced that it was strengthening its corporate governance with the
establishment of an audit committee within its board of directors, and appointing three new
independent directors. In addition, recent research shows that the compensation of top executives in
Chinese companies is closely related to prior and current financial performance of the firm.
8. How does corporate governance foster ethical strategic decisions and how important is
this to top-level executives?
Governance mechanisms focus on the control of managerial decisions to ensure that the interest of
shareholders, the most important stakeholder, will be served. But shareholders are just one
stakeholder along with product market stakeholders (e.g., customers, suppliers, and host communities)
and organizational stakeholders (e.g., managerial and nonmanagerial employees). These stakeholders
are important as well. Therefore, at least the minimal interests or needs of all stakeholders must be
satisfied through the firm's actions. Otherwise, dissatisfied stakeholders will withdraw their support
from one firm and provide it to another (e.g., employees will exit and seek another employer,
customers seek other vendors, etc.). Some believe that ethically responsible companies design and use
governance mechanisms to ensure that the interests of all stakeholders are served.Top-level executives
are monitored by the board of directors. All corporate stakeholders are vulnerable to unethical
behaviors by the firm. If the image of the firm is tarnished, the image of customers, suppliers,
shareholders, and board members is also tarnished. Top-level managers, as the agents who have been
hired to make decisions that are in shareholders' best interests, are ultimately responsible for the
development and support of an organizational culture that allows unethical decisions and behaviors.
The board of directors has the power and responsibility to enforce this expectation.The decisions and
actions of a corporation's board of directors can be an effective deterrent to unethical behaviors. The
board has the power to hold top managers accountable for unethical actions as they can hire and fire
these managers. Thus, the board of directors, which holds a position above the firm's highest-level
managers, holds considerable power over top-level executives and can set and enforce standards for
ethical behaviors within the organization.
Case Scenario 1: Abramson's Jewelers. Abramson's Jewelers has established a strong niche
market in the upscale jewelry store segment. Abramson's was founded in 1871, and its current
single-store location is owned and operated by John Wickersham, who bought the firm from its
namesake founders in 1985. Over the last 15 years, Mr. Wickersham has narrowed the
company's product offering considerably to focus only on high-end watches like Rolex and
Piaget, custom jewelry, and estate jewelry. Mr. Wickersham stresses that this is an appropriate
focus for his business since each of the products lends itself to relationship selling, and price
rarely comes into the discussion. Despite the narrower offering, Abramson's floor space has
doubled, and clients are intensely loyal to the good taste, design skills, and personal service level
provided by Mr. Wickersham. After evaluating several expansion options, Mr. Wickersham has
decided to open another store in a neighboring city. While it is likely that some of his existing
customers may begin doing business at the other location, thus lowering sales volume at the
original store, Mr. Wickersham sees this as a desirable increase in the level of service and
convenience he can provide his existing clientele. At the same time, he believes that he will be
able to grow the overall business faster with two locations. He has identified another reputable
gemologist, Jill Diamond, to run the other store and is now considering how to compensate her.
-(Refer to Case Scenario 1)
1. What are the advantages and disadvantages of paying the new manager primarily cash
pay?
The best answers will note, assuming Ms. Diamond has an interest in seeing the store do well, that
cashpay may let her focus on maximizing the level of service provided to the extant customer base. In
terms of drawbacks, there is little incentive for Ms. Diamond to grow the business, which is what Mr.
Wickersham is aiming for.
2. What are the advantages and disadvantages of paying the new manager primarily on
new store sales growth?
The best answers here will contrast the benefits of cash to incentive pay, and note that incentive pay
will focus Ms. Diamond's efforts on increasing the customer base and the dollar sales per customer. A
potential negative consequence would be for her to lose sight of customer service (retention), since
she is working so hard to generate new sales.
3. What compensation structure would you recommend?
Answers should begin by suggesting that both components of pay are important and that some balance
needs to be struck between the two. A fair level of cash pay can motivate Ms.Diamond to keep the
existing business running at a high service level for existing customers. An added incentive pay
component can provide her with additional motivation to grow the business. The best answers here
will take the concept of incentive pay further to suggest that Mr. Wickersham can establish a number
of objectives for which the incentive pay can be earned. Such a balanced scorecard approach provides
Ms. Diamond with the incentive to manage the whole store, and not one or two particular dimensions
of it.
Case Scenario 2: Raptec. Raptec operates in three principal business segments: Direct Attached
Storage ("DAS"), Storage Networking Solutions ("SNS") and Software. These hardware and
software products are found in high-performance networks, servers, workstations and desktops
from the world's leading OEMs, and are sold through distribution channels to Internet service
providers, enterprises, medium and small businesses and consumers. Since the time it went
public, Raptec has experienced rapid growth and consistently profitable operations. In early
2002, Raptec announced its plan to spin-off the software segment, subsequently incorporated as
Axio, Inc., in the form of a fully independent and separate company. Software was Raptec's
most profitable and fastest growing segment. By mid-2002 Raptec had completed the initial
public offering of approximately 15% of Axio's stock, and then distributed the remaining Axio
stock to Raptec's stockholders in a tax-free distribution. Axio's family of products includes
category leaders in CD/DVD burning, digital photography, and digital video. Axio's new
management team is comprised of: Lex Luthor, CEO, and previously the President of New
Business Development for Universal Studios Recreation Group; Karal Kool, COO, and
previously General Manager of Raptec's OEM Solutions Group; and R. Elliot Maxter, CFO,
and previously corporate controller for Raptec. The interim four-member board of directors is
currently comprised of Raptec senior officers, but the terms of the public offering require them
to step down in two months. Thus, Axio will need to construct a new board, which in turn will
be responsible for overseeing Axio's management and their compensation. -(Refer to Case
Scenario 2)
1. What characteristics will you look for in appointing new board members?
The best answers will begin by noting that Axio should probably look outside the firm for new
directors,to provide some balance of power and accountability. With this foundation, students can
then debate what aspects of the firm would benefit most through external advice and counsel: legal,
management consulting, finance, emerging markets, key customers, people with experience in rapidly
growing firms, etc. These areas suggest the type of individuals and what experience and contacts
should be sought out as new directors.
2. Develop arguments as to why and why not Lex Luthor should be appointed as
chairperson of the new board.
The case where a CEO also occupies the chairperson position (referred to as CEO duality) is common,
but increasingly controversial. The best answers will note that there are certain trade-offs between
this choice, and hence no easy answer. On the one hand, a CEO/chairperson may be able to execute
strategies quicker given his or her added power and knowledge accrued from the role as CEO. Such
nimble decisiveness may be very important for firms in fast cycle industries like software. On the
other hand, duality dulls the clean distinction between ownership (through board representation) and
managerial control, and increases managerial power and hence the threat of opportunistic behavior.
3. How should the board design the executive compensation scheme for Luthor, Kool, and
Maxter?
Most CEOs in fast-cycle industries also serve as chairperson. This means that the power balance and
incentive alignment may have to be achieved through the level and structure of compensation.
Regardless, speaking only to CEO compensation, the level of cash compensation would need to be set
high enough to attract a talented manager. Students may also agree that the bulk of compensation
should come in the form of bonuses, stock options, or other long-term incentive pay since such a
structure will reward the CEO for increases in shareholder value. This is the essential objective of
compensation, the alignment of executive and shareholder interests. The students should make the
point that a compensation plan (such as bonuses and other incentive pay) that is based on short-term
outcomes encourages the top managers to neglect the long-run welfare of the firm and its
shareholders, and even to manipulate the company's financial results.
Quiz 11: Organizational Structure and Controls
1. Discuss the difference between strategic and financial controls.
Strategic Controls: Subjective criteria
- Concerned with examining the fit between:
- what the firm might do (opportunities in its external environment).
- what the firm can do (competitive advantages).
- Evaluate the degree to which the firm focuses on the requirements to implement its strategy
Financial Controls: Objective criteria
- Accounting-based measures
- Return on investment
- Return on assets
- Market-based measures
- Economic Value Added (EVA)
Strategic controls (largely subjective criteria) and financial controls (largely objective criteria) are the
two types of organizational controls used to implement a strategy. Both controls are critical, although
their degree of emphasis varies based on individual matches between strategy and structure.
2. Describe the three major types of organizational structure and their appropriate use.
Organizational structure specifies the firm’s formal reporting relationships, procedures, controls, and
authority and decision making processes. Essentially, organizational structure details the work to be
done in a firm and how that work is to be accomplished. Organizational controls guide the use
of strategy, indicate how to compare actual and expected results, and suggest actions to take to
improve performance when it falls below expectations. A proper match between strategy and structure
can lead to a competitive advantage anticipate the need for structural change and quickly modify
structure to better accommodate the firm’s strategy when evidence calls for that action.
3. Discuss the organizational structures used to implement the different business-level
strategies.
4. Define the three major dimensions of organizational structure: specialization, centralization,
and formalization. How do these dimensions vary in organizations implementing the cost-leadership,
differentiation, and the cost-leadership/differentiation strategies?
5. Discuss the organizational structures used to implement corporate-level strategies.
6. Describe the organizational structure associated with a firm that pursues an unrelated
diversification strategy.
7. Describe the organizational structures used to implement the three international strategies.
8. Describe the organizational structures used to implement cooperative strategies, giving
attention to the role of the strategic center firm.

Case Scenario 1: Compliance, Inc. Compliance, Inc., (CI) conducts clinical human and animal
trials for the pharmaceutical and biotechnology industries. Revenues are split evenly between
early and late drug development services. While the bulk of its business is conducted in Europe
and the U.S. (10 and 17 subsidiaries, respectively), CI also has subsidiaries in Africa, Latin
America, Asia, and Australia. Historically CI operated under a multidomestic strategy, owing to
the fact that the clinical testing industry was geographically fragmented to meet the diverse
needs of the many strong local pharmaceutical companies and distinct regulatory environments.
CI's organizational structure truly reflected the autonomous character of each country's
businesses. Many of the country managers have been with CI for over a decade, and have a
great deal of discretion over the activities of their home-market businesses. However,
globalization of the regulatory environment (both global and local standards), globalization of
the biotechnology firms (increasing the geographic scope of their operations), and tremendous
consolidation in the pharmaceutical industry (reducing the number of pharmaceutical industry
participants to only a handful of major global companies) caused CI to question its
multidomestic strategy. Consequently, the firm has begun its transition to a transnational
strategy. -(Refer to Case Scenario 1)
1. What type of organizational structure was likely to have been in place under CI's
multidomestic strategy?
2. What type of organizational structure will likely be needed for its transnational strategy?
What impact will this have on the location of particular value chain activities?
3. What obstacles is CI likely to encounter as it attempts to change its structure to support the
transnational strategy?

Case Scenario 2: Palmetto. Palmetto was an early pioneer of personal data assistants (PDAs)
and dominates that market space (in terms of market share) with its core product, the Palmetto
Pidgy. Because this product category was entirely new to the market, Palmetto had to internally
develop the hardware and software sides of the business, and today is both a manufacturer of
PDAs and a programmer and licensor of its PDA operating system software. Recently however,
the hand-held device maker's performance has taken a dive as a result of slumping sales and
costly inventory problems. Palmetto has also had difficulty coordinating its software and
hardware businesses, in part due to the near absence of a coherent structure and the differing
economics underlying the two. Specifically, hardware for PDAs is increasingly a cost-based
business, while software is a highly differentiated one. While Palmetto is doing pretty well in
both businesses, its own resource base does not allow it to compete any differently than that
proscribed for other industry participants (i.e., competes on cost with hardware and features
with software). In addition to these fundamental differences, new large entrants are entering
both the equipment (e.g., Sony) and software (e.g., Microsoft) sides of its business, putting
further pressure on margins. Management has decided that it is unable to focus on the
complexities of each of these businesses so it is opting to break Palmetto into two separate,
independent public companies - Pal, Inc. will be devoted to hardware and Mettolink, Inc. will be
devoted to software. -(Refer to Case Scenario 2)
1. What basic structural form would you recommend for both Pal and Mettolink? What must
each firm be careful to avoid with this structure?
2. How would the implementation of this structure differ for Pal?
3. How would the implementation of this structure differ for Mettolink?
Quiz 12: Strategic Leadership
https://quizlet.com/85357941/strategic-management-ch-12-essay-flash-cards/
1. What is strategic leadership, who has primary responsibility for strategic leadership,
and what are the five key strategic leadership actions?
Strategic leadership is the ability to anticipate, envision, maintain flexibility, and empower others to
create strategic change. The CEO has primary responsibility for strategic leadership, which is shared
with the board of directors, the top management team and divisional general managers. The five key
strategic leadership actions are: determining a strategic direction, effectively managing the firm's
resource portfolio, sustaining an effective organizational culture, emphasizing ethical practices, and
establishing balanced organizational controls.
2. What is a top management team, and how does it affect a firm's performance and its
abilities to innovate and design and implement effective strategic changes?
The top management team is composed of the key managers in the organization who are responsible
for selecting and implementing the firm's strategy. Typically, the top management team includes all
officers of the firm (defined by the title of vice president or above) and/or those who serve as a
member of the board of directors. Team characteristics have been shown to affect the strategy of the
organization. A heterogeneous top management team is composed of individuals with varied
functional backgrounds, experiences, and education. A homogeneous team's members are similar to
one another in characteristics and experiences. A heterogeneous team is more likely to formulate an
effective strategy because of its varied expertise and knowledge. Additionally, heterogeneous top
management teams have been shown to positively affect performance. In particular, heterogeneous
teams positively affect innovation and strategic change in firms. But, heterogeneous teams are less
cohesive than homogeneous teams because of communication difficulties, and it is more difficult for
heterogeneous teams to implement strategies. Consequently, a heterogeneous top management team
must be managed effectively to use the diversity in a positive way.
3. Discuss how the managerial succession process and the composition of the top
management team interact to affect strategy.
Internal labor markets represent the opportunities for employees to take managerial positions
(including the position of CEO) within a firm. The external labor market is the collection of career
opportunities for managers in firms outside of the one for which they currently work. CEOs may be
selected from internal or external candidates. Internal CEO selection is preferred by employees and by
those who wish the firm to continue in its present strategies. External CEO succession is considered a
sign that the board of directors wants change. Internal CEOs are less likely to seek change in the
firm's strategy than external CEOs. It is important to note that the source of the CEO (from the
internal or external labor market) and the top management team's composition interact to affect the
likelihood of strategic change.
If a firm hires a new internal CEO and has a homogeneous top management team, it
is unlikely that the firm's strategy will change. If the firm employs a new internal CEO but has a
heterogeneous top management team, it will probably continue the current strategy, but innovation
will be encouraged. If the top management team is homogeneous, but an external CEO is chosen, the
situation will be ambiguous. Finally, if the top management team is heterogeneous and an external
CEO is chosen, strategic change is likely.
4. Define human capital and its importance to the firm's success.
Human capital represents the knowledge and skills of the firm's entire workforce.
Effective strategic leaders view human capital as a capital resource that requires investment rather
than as a cost to be minimized. It is thought that people are the organization's only truly sustainable
source of competitive advantage. So, effective human resource management practices are necessary to
successfully select and use people to attain the firm's goals. Not only must future leaders be trained,
but the entire workforce must be able to learn continuously to build skills and knowledge that lead
toward innovation. Layoffs can be disastrous because they strip skills and knowledge from the firm,
leaving remaining employees unable to perform their tasks effectively.
5. What is organizational culture? What must strategic leaders do to develop and sustain
an effective organizational culture?
Organizational culture is the set of ideologies, symbols, and core values that is shared throughout the
organization and that influences the way the firm conducts its business. An organization's culture can
be a source of competitive advantage. It is more difficult to change a firm's culture than to sustain it.
But effective strategic leadership recognizes when a change in a firm's culture is necessary.
Incremental changes to the firm's culture are typically used to implement strategies. Sometimes
radical changes are used to support strategies that differ from the firm's historical pattern. Shaping and
reinforcing change in an organization's culture require communication and problem solving, selection
processes that find people with the right values, effective performance appraisals focused on goals
reflecting the new culture, and reward systems that reward behaviors reflecting the new core values.
Change occurs only when it is actively supported by the CEO, other top managers, and middle
management.
6. As a strategic leader, what actions could you take to establish and emphasize ethical
practices in your firm?
Ethical practices should be institutionalized within the organization by being an integral part of the
organizational culture. Sustaining an effective organizational culture is one of the key leadership
actions and one aspect of an effective culture is that it promotes ethical behavior in the organization.
Strategic leaders also develop explicit codes of conduct and provide ethics training to disseminate
those codes. Examples of specific actions taken by strategic leaders to develop an ethical
organizational culture include: (1) establishing and communicating specific goals to describe the
firm's ethical standards (e.g., developing and disseminating a code of conduct), (2) continuously
revising and updating the code of conduct, based on inputs from people throughout the firm and from
other stakeholders (e.g., customers and suppliers), (3) disseminating the code of conduct to all
stakeholders to inform them of the firm's ethical standards and practices, (4) developing and
implementing methods and procedures to use in achieving the firm's ethical standards (e.g., use of
internal auditing practices that are consistent with the standards), (5) creating and using explicit
reward systems that recognize acts of courage (e.g., rewarding those who use proper channels and
procedures to report observed wrongdoing), and (6) creating a work environment in which all people
are treated with dignity.
7. What are organizational controls? Why are strategic controls and financial controls
important aspects of the strategic management process?
Organizational controls are the formal, information-based procedures used by managers to maintain or
alter patterns in organizational activities. Controls provide the parameters within which strategies are
to be implemented, as well as forming guidelines for corrective actions when adjustments are
required. There are two main types of controls: financial and strategic. Financial controls focus on
short-term financial outcomes. Strategic controls focus on the content of strategic actions. Financial
controls give feedback about the outcomes of past actions. Strategic controls focus on the drivers of
the firm's future performance. Emphasizing either financial or strategic controls has important
implications for the strategic management process. For example, emphasizing financial controls often
produces more short-term and risk-averse managerial actions because financial outcomes may be
caused by events beyond the managers' direct control. In contrast, strategic control encourages lower-
level managers to make decisions that incorporate moderate and acceptable levels of risk because
outcomes are shared between the business-level executives making strategic proposals and the
corporate-level executives evaluating them.

Case Scenario 1: The Walt Disney Company The Walt Disney Company was founded as a
cartoon studio in 1923 by Walt Disney and his brother Roy with a $500 loan from an uncle. In
the early 1920s, cartoonist Walt Disney visited New York to pitch his idea for a cartoon rabbit
called Waldo. During that trip, through a complicated series of events, Disney lost the rights to
develop Waldo. On the train-ride back to California he spoke with his wife about the
importance of coming home with some alternative character. "I can't come back to our office
and tell them I've lost Waldo," he bemoaned. This hardship inspired Disney to develop a new
character, Mickey Mouse, and release the world's first fully-synchronized sound cartoon,
"Steamboat Willie" (starring, of course, Mickey Mouse). Disney's creative genius was now
coupled with a fierce instinct to protect and control his creative output. Never again would he
lose "Waldo." Consequently, the Walt Disney Company was pushed by Walt to tirelessly create
timeless and universal entertainment, consistently innovate and take risks to deliver that
entertainment, stress a vision of being the provider of choice of quality family entertainment,
and maintain rigorous control over the quality of customers' experiences with Disney products
and its image. Such a personal passion for control led the Walt Disney Company into theme
parks because Disney did not want Mickey's reputation sullied by the dirty, cheap theme parks
that littered the land during those days. All films had to be new and of the highest quality
animation (taking a minimum of five years to create, including hand-painted backgrounds);
sequel films were not tolerated. Walt's vision and risk taking propensity led him in the early
1960s to buy 43,000 acres in Florida (now Walt Disney World), betting the company's future on
a high-risk, uncertain venture. Amidst such a flurry of activity, Walt Disney died just before
Christmas 1966, and the company was literally stopped dead in its tracks. Walt Disney's
blueprint was being followed to the letter, but no further (Walt Disney World opened in 1971).
No "new" creations were undertaken until 1982, when the company finally launched such
businesses as the Disney Channel, Touchstone, and their home video business. Had it not been
for the appointment of Michael Eisner as Disney's new CEO in 1984, the company would likely
not have survived its perilous financial situation and stifled creativity. Eisner returned the
company to its roots of family entertainment and values of quality, fairness, creativity,
entrepreneurialism, and teamwork. -(Refer to Case Scenario 1)
1. What value-creating legacy did Walt Disney leave to the Walt Disney Company?
2. To what extent had the Walt Disney Company become a reflection of Walt up to the time that
he died in 1966?
3. Why do you think the Walt Disney Company had so much difficulty being innovative in the
decades following Walt's death?

Case Scenario 2: Yepsen Timber Farms, Inc. Yepsen Timber Farms, Inc., (YTF) was started
around 1933 by Danish immigrants. The firm's primary operations were timber harvesting on
several thousand acres in Oregon acquired in part under the Homestead Act, and in part
through direct purchase. The firm was founded, initially as a partnership, between brothers
Mogens and John (Jack) Yepsen. The Yepson brothers were among the first four graduates at
Oregon Agricultural College (now Oregon State University), worked for the forest service and
private industry in Oregon for a number of years, then quit their respective jobs to manage the
forest they had been developing for a number of years. While timber is considered a low-tech
type business, Mogens and Jack were very innovative from the standpoint that they established
"tree farms," that is, harvesting then replanting acreage so that it would yield timber on a
sustainable basis. At the time, and in certain parts of the world to this day, timber lands were
typically "clear cut" where all the trees were stripped from a property, then the timber
harvester simply moved to another parcel. This practice left thousands of acres barren, and
often damaged valuable animal habitats and watershed. The brothers also introduced hybrid
Pine and Douglas Fir trees that grew considerably faster than the native forest stock. These
factors allowed them to grow trees that would be ready for market in 25 years, about half the
time of that required to grow native trees. The brothers' idea about regeneration, care for the
environment, and hybridization defined the YTF business. Never would land be harvested
faster than it could replenish itself, or in a manner that threatened habitats or watersheds.
Eventually, Mogens and Jack passed on and their only surviving children, Marjorie, Mary
Jane, Burton, and Betty inherited the property. Two of these heirs took a strong interest in
further building the portfolio of Oregon properties, and also converted the holdings to an S-
Corp. to allow for the distribution of ownership and earnings to their own children. Under their
guidance, YTF was tremendously successful and garnered much community acclaim for its
sustainable farming practices. Now, the four siblings are in their 70s and few of their children
have expressed much interest in managing the extensive portfolio of timber holdings. Among
those that have expressed an interest, some are very knowledgeable about forestry, while others
have a track record of incompetence and self-promotion. At the same time, ownership is now
spread among some 40 children, nieces, nephews, and grandchildren of the four siblings. Many
of these individuals' only interest in YTF is the annual dividend check they receive. -(Refer to
Case Scenario 2)
1. What culture did Mogens and Jack nurture in YTF?
2. How important is this culture to the future success of YTF?
3. What must be done to continue the viability of YTF as a sustainable timber farm?

Case Scenario 3: Zachary, Wesley & Partners. Zachary, Wesley & Partners (ZW&P) is a
leveraged buyout (LBO) firm that specializes in friendly buyouts of mid-sized U.S. retailing and
manufacturing firms. ZW&P shuns turnarounds and hostile takeovers; its typical deals retain
the existing management team and provide extensive funding for what is perceived to be an
already sound strategy. It focuses on this type of firm because the partners have good contacts
in retailing and manufacturing and they are typically able to avoid bidding wars when the LBO
is negotiated. The firm has been immensely profitable over the years, in part due to the very
extensive and selective due diligence process used to winnow down the list of prospective
targets. Fewer than one out of one hundred candidates are even approached, and only a fraction
of these passes further screens in the LBO negotiations. The resulting profitability has, in turn,
given ZW&P a strong reputation in the financial community for successful deals, and among
managers for being able to put together needed financing with good business plans. -(Refer to
Case Scenario 3)
1. What are this firm's core resources and capabilities?
2. Where are these core resources likely to be located in the firm?
3. What factors may threaten the ability of ZW&P's resources and capabilities to generate
continued success?

Quiz 13: Strategic Entrepreneurship


1. Define the three types of innovative activity. Which is the most critical activity for U.S.
firms?
Firms engage in three types of innovative activity: (1) invention, which is the act of creating a new
good or process, (2) innovation, or the process of creating a commercial product from an invention,
and (3) imitation, which is the adoption of similar innovations by different firms. Invention brings
something new into being while innovation brings something new into use.
2. What is the importance of international entrepreneurship?
International entrepreneurship, or the process of identifying and exploiting entrepreneurial
opportunities outside the firm’s domestic markets, has become important to firms around the globe.
Evidence suggests that firms capable of effectively engaging in international entrepreneurship
outperform those competing only in their domestic markets.
3. Describe the three strategic approaches used to produce and manage innovation:
internal corporate venturing, cooperative strategies, and acquisitions.
Three basic approaches are used to produce innovation:
(1) internal innovation, which involves R&D and forming internal corporate ventures, (2) cooperative
strategies such as strategic alliances, and (3) acquisitions. Autonomous strategic behavior and induced
strategic behavior are the two forms of internal corporate venturing. Autonomous strategic behavior is
a bottom-up process through which a product champion facilitates the commercialization of an
innovative good or service. Induced strategic behavior is a top-down process in which a firm’s current
strategy and structure facilitate the development and implementation of product or process
innovations. Thus, induced strategic behavior is driven by the organization’s current corporate
strategy and structure while autonomous strategic behavior can result in a change to the firm’s current
strategy and structure arrangements.
4. Discuss the differences between autonomous strategic behavior and induced strategic
behavior.

Induced Strategic Behavior Autonomous Strategic


Behavior

Most large companies employ this. Employees are allowed to think


Planned form of influence upon the creatively about innovation
workforce to come up with innovative outside of their present
thinking around their present product. products. They can think about
products that they like to create

Nature of screening Formal screening managers define a market


process diverges from the
organization's strategy

activation of the manager defines the market that is in line Informal network that assesses
strategic decision with the org's strategy new ideas
process

Type of innovation incremental to present products major - whole new product


lines

Nature of consistent with organizational work flow Autonomous departs from work
communication in early stages of the decision

Major Actors formal as prescribed by the organization informal network and furthered
by a so-called "champion)

Decision Roles roles and responsibilities are well roles and responsibilities are
defined loosely defined in early stages
but become more defined as the
project progresses

Implications for strategic alternatives are considered and commitment to a particular


Strategy a commitment to a particular strategy strategy emerges during the
evolves early stages as the project
progresses through sponsorship
of the "Product Champion"

5. Discuss the methods an organization can use to facilitate cross-functional integration.

6. Discuss the potential benefits and disadvantages of innovation through cooperative


strategies.

7. Discuss the benefits and risks of acquiring another firm to gain access to innovations.

Case Scenario 1: Fear Not. Wim Vijkland was trained as an engineer in the Netherlands and,
after college, worked several years in the Chinese operations of Philips Electronics and then
Unilever. Between employers he returned home for several years to complete an MBA from
Tiburg University in the southern Netherlands. His work gave him hands-on experience with
overseas production, and rich sets of contacts in Mainland China and distribution channels in
Europe and the U.S. Wim has noted that many small and mid-sized European and U.S.
manufacturers are interested in and would benefit from the low-cost Chinese production
environment. Contrary to external stereotypes, he also believes that a Chinese factory can
produce products that meet the most demanding technical and quality specifications met by
manufacturers in more developed economies. At the same time, Vijkland understands that
"foreigners" are generally reluctant to manufacture precision products in China for fear that
the underlying proprietary technologies will be bootlegged and sold to competitors or outright
copied. In an attempt to capitalize on this opportunity, Wim quit his job with Unilever and
entered into a partnership with Sulin "Cathy" Liu, a local Beijing entrepreneur with whom
Wim has worked extensively in the past. Cathy has a Ph.D. in physics from CalTech in
California and an MBA from Hong Kong University of Science and Technology. They have
dubbed their partnership FearNot, and organized it as a limited liability corporation (LLC).
Their plan is to set up duty-free manufacturing zones in which they develop mini-factories that
operate under their ownership and production guidance, while at the same time creating a
firewall between the clients' proprietary production processes and the open Chinese market. It
is the partners' hope that this combination of intellectual property protection and low-cost
overseas production will provide U.S. and European firms an incentive to enlist FearNot's
services. -(Refer to Case Scenario 1)
1. What resources do Wim and Cathy bring to their venture?
2. What resources appear to be missing?
3. What should FearNot focus on in its first months of operation?

Case Scenario 2: Compliance, Inc. Compliance, Inc. (CI) conducts clinical human and animal
trials for the pharmaceutical and biotechnology industries. Revenues are split evenly between
early and late drug development services and the firm is a leader in the laboratory technologies
needed for such testing. One of CI's internal quality managers, Sharon Kline, has approached
the CEO with a new business proposal. She would like to see the firm take one of its in-house
software programs and develop it as a leading-edge commercial product for three specific target
markets - medical care providers, payers of medical care, like insurance companies, and
suppliers to medical care providers, like pharmaceutical companies. The features of the
software are easy to use and include electronic distribution, data harvesting, and robust
reporting capabilities. With this software Sharon believes that medical care providers will be
able to collect data to market to and negotiate contracts with payers or employers, profile
performance of individual physicians or practice sites, identify best clinical practices, generate
reports that satisfy regulatory or accreditation requirements for provider sites, and supply
professional societies with data for influencing payer and government policies. Another target
market, insurance companies and other medical services payers, will be able to use the software
to profile performance of individual physicians or practice sites, identify best clinical practices,
generate reports that satisfy regulatory or accreditation requirements, and collect data to
market to and negotiate contracts with employers. Finally, the software will allow suppliers to
medical care providers to assess how products perform compared to competitor products,
assess outcomes in real-world compared to clinical trial settings, obtain information on
provider-specific practice patterns, determine whether products are being used correctly, get
"face-time" with physicians and HMOs, obtain information on product switching behavior,
offer providers a value-added service, and meet FDA post-marketing surveillance requirements.
CI has never launched such a product before and, even if successful, software is a very different
product than the clinical trials services it provides now. The CEO must determine how to build
and manage this new business for CI. -(Refer to Case Scenario 2)
1. Does CI's launch of the software product better fit the notion of autonomous or induced
strategic behavior?
2. Where does it appear that autonomous strategic behavior ends and induced strategic behavior
begins in the software situation at CI?
3. What actions would you recommend that the CEO undertake to ensure the success of the
software venture?

Case Scenario 3: Bunnywac. Bunnywac is a global producer and seller of batteries for consumer
electronics, and competes primarily by providing battery products equal in performance at a
lower price. The worldwide battery industry suffers from issues of overcapacity and
commoditization, brand segmentation and proliferation, the growing strength of global
retailers, and the low-cost threat of new entrants from Asia. Bunnywac has grown quickly into
one of the leading players in the battery industry primary through horizontal acquisitions, and
is now counted among the top four companies in North and Latin America. Its presence in
Europe and Latin America is negligible. A key factor in Bunnywac's rapid growth is its
technology outsourcing arrangement with Mats. Mats is one of Japan's largest technology
holding companies and Bunnywac's core battery technology is licensed from Mats. Bunnywac's
license with Mats expires soon and it is concerned that Mats will not renew it, or will renew it
only for a substantial price premium. Consequently, Bunnywac's CEO is exploring the
possibility of developing its core technology in-house. -(Refer to Case Scenario 3)
1. Does Bunnywac's development of its core battery technology better fit the notion of
autonomous or induced strategic behavior?
2. What aspects of Bunnywac's development of its core battery technology do not fit the notion
of strategic behavior that you identified above?
3. How will Bunnywac's structure likely need to change if it does begin developing its core
battery technologies in-house?

You might also like